Download as pdf or txt
Download as pdf or txt
You are on page 1of 86

Chemistry

Unique Practice Questions


(Collection of JEE Main 2019-22)
For More Material Join: @JEEAdvanced_2024
Unique Practice Questions Chemistry

Corporate Office : Aakash Tower, 8, Pusa Road, New Delhi-110005


Ph.: 011-47623456

Unique Practice Questions (Chemistry)


1. Consider the following reaction:

The product ‘X’ is used

(1) In acid base titration as an indicator

(2) In protein estimation as an alternative to ninhydrin

(3) In laboratory test for phenols

(4) As food grade colourant

2. The electron gain enthalpy (in kJ/mol) of fluorine, chlorine, bromine and iodine, respectively, are :

(1) –296, –325, –333 and –349 (2) –333, –325, –349 and –296

(3) –349, –333, –325 and –296 (4) –333, –349, –325 and –296

3. The number of chiral carbons in chloramphenicol is ___________.

4. The flocculation value of HCl for arsenic sulphide sol. is 30 m mol L–1. If H2SO4 is used for the flocculation of
arsenic sulphide, the amount, in grams, of H2SO4 in 250 ml required for the above purpose is ______.

(molecular mass of H2SO4 = 98 g/mol)

5. The number of sp2 hybridised carbons present in “Aspartame” is ______.

6. As per Hardy-Schulze formulation, the flocculation values of the following for ferric hydroxide sol are in the
order

(1) AlCl3 > K3[Fe(CN)6] > K2CrO4 > KBr = KNO3 (2) K3 [Fe(CN)6] < K2CrO4 < KBr = KNO3 = AlCl3

(3) K3[Fe(CN)6] > AlCl3 > K2CrO4 > KBr > KNO3 (4) K3[Fe(CN)6] < K2CrO4 < AlCl3 < KBr < KNO3

Corporate Office : Aakash Tower, 8, Pusa Road, New Delhi-110005 [1]

For More Material Join: @JEEAdvanced_2024


For More Material Join: @JEEAdvanced_2024
Unique Practice Questions Chemistry

7. The number of chiral centres in penicillin is _____________.

8. For the following Assertion and Reason, the correct option is

Assertion : For hydrogenation reactions, the catalytic activity increases from Group 5 to Group 11 metals
with maximum activity shown by Group 7-9 elements.

Reason : The reactants are most strongly adsorbed on group 7-9 elements.

(1) Both assertion and reason are true and the reason is the correct explanation for the assertion.

(2) Both assertion and reason are false.

(3) The assertion is true, but the reason is false.

(4) Both assertion and reason are true but the reason is not the correct explanation for the assertion.

9. White phosphorus on reaction with concentrated NaOH solution in an inert atmosphere of CO 2 gives phosphine
and compound (X). (X) on acidification with HCl gives compound (Y). The basicity of compound (Y) is

(1) 3 (2) 2

(3) 4 (4) 1

10. Complexes (ML5) of metals Ni and Fe have ideal square pyramidal and trigonal bipyramidal geometries,
respectively. The sum of the 90°, 120° and 180° L-M-L angles in the two complexes is __________.

11. The electronic configurations of bivalent europium and trivalent cerium are

(atomic number : Xe = 54, Ce = 58, Eu = 63)

(1) [Xe] 4f7 and [Xe] 4f1 (2) [Xe] 4f7 6s2 and [Xe] 4f2 6s2

(3) [Xe] 4f2 and [Xe] 4f7 (4) [Xe] 4f4 and [Xe] 4f9

12. A chemist has 4 samples of artificial sweetener A, B, C and D. To identify these samples, he performed certain
experiments and noted the following observations :

(i) A and D both form blue-violet colour with ninhydrin.

(ii) Lassaigne extract of C gives positive AgNO3 test and negative Fe4[Fe(CN)6]3 test.

(iii) Lassaigne extract of B and D gives positive sodium nitroprusside test.

Based on these observations which option is correct?

(1) A : Aspartame; B : Alitame;

C : Saccharin; D : Sucralose

Corporate Office : Aakash Tower, 8, Pusa Road, New Delhi-110005 [2]

For More Material Join: @JEEAdvanced_2024


For More Material Join: @JEEAdvanced_2024
Unique Practice Questions Chemistry

(2) A : Saccharin; B : Alitame;

C : Sucralose; D : Aspartame

(3) A : Alitame; B : Saccharin;

C : Aspartame; D : Sucralose

(4) A : Aspartame; B : Saccharin;

C : Sucralose; D : Alitame

13. Which of these will produce the highest yield in Friedel Crafts reaction?

(1) (2)

(3) (4)

14. The mass percentage of nitrogen in histamine is _____.

15. A, B and C are three biomolecules. The results of the tests performed on them are given below

A, B and C are respectively :

(1) A = Lactose, B = Fructose, C = Alanine (2) A = Lactose, B = Glucose, C = Alanine

(3) A = Glucose, B = Fructose, C = Albumin (4) A = Lactose, B = Glucose, C = Albumin

16. Biochemical Oxygen Demand (BOD) is the amount of oxygen required (in ppm)

(1) By bacteria to break-down organic waste in a certain volume of a water sample

(2) For sustaining life in a water body

(3) By anaerobic bacteria to break down inorganic waste present in a water body

(4) For the photochemical break down of waste present in 1 m3 volume of a water body

Corporate Office : Aakash Tower, 8, Pusa Road, New Delhi-110005 [3]

For More Material Join: @JEEAdvanced_2024


For More Material Join: @JEEAdvanced_2024
Unique Practice Questions Chemistry

17. On heating compound (A) gives a gas (B) which is a constituent of air. This gas when treated with H 2 in the
presence of a catalyst gives another gas (C) which is basic in nature. (A) should not be:

(1) Pb(NO3)2 (2) (NH4)2Cr2O7

(3) NH4NO2 (4) NaN3

18. While titrating dilute HCl solution with aqueous NaOH, which of the following will not be required?

(1) Pipette and distilled water (2) Clamp and phenolphthalein

(3) Burette and porcelain tile (4) Bunsen burner and measuring cylinder

19. The statement that is not true about ozone is

(1) In the stratosphere, CFCs release chlorine free radicals (Cl) which reacts with O3 to give chlorine dioxide
radicals

(2) It is a toxic gas and its reaction with NO gives NO2.

(3) In the atmosphere, it is depleted by CFCs

(4) In the stratosphere, it forms a protective shield against UV radiation

20. If you spill a chemical toilet cleaning liquid on your hand, your first aid would be

(1) aqueous NaOH (2) aqueous NaHCO3

(3) aqueous NH3 (4) vinegar

21. The correct observation in the following reactions is :

(1) Gives no colour (2) Formation of red colour

(3) Formation of violet colour (4) Formation of blue colour

22. The atomic number of the element unnilennium is

(1) 109 (2) 119

(3) 102 (4) 108

23. The antifertility drug “Novestrol” can react with

(1) Br2/water; ZnCl2/HCl; FeCl3 (2) ZnCl2/HCl; FeCl3; Alcoholic HCN

(3) Br2/water; ZnCl2/HCl; NaOCl (4) Alcoholic HCN; NaOCl; ZnCl2/HCl

Corporate Office : Aakash Tower, 8, Pusa Road, New Delhi-110005 [4]

For More Material Join: @JEEAdvanced_2024


For More Material Join: @JEEAdvanced_2024
Unique Practice Questions Chemistry

24. Thermal power plants can lead to

(1) Eutrophication (2) Ozone layer depletion

(3) Blue baby syndrome (4) Acid rain

25. The incorrect statement(s) among (a) – (d) regarding acid rain is (are)

(a) It can corrode water pipes.

(b) It can damage structures made up of stone.

(c) It cannot cause respiratory ailments in animals.

(d) It is not harmful for trees.

(1) (a), (c) and (d) (2) (a), (b) and (d)

(3) (c) and (d) (4) (c) only

26. Match the following drugs with their therapeutic actions

Drug Therapeutic Action

(i) Ranitidine (a) Antidepressant

(ii) Nardil (b) Antibiotic

(Phenelzine)

(iii) Chloramphenicol (c) Antihistamine

(iv) Dimetane (d) Antacid

(Brompheniramine)

(e) Analgesic

(1) (i)-(d); (ii)-(c); (iii)-(a); (iv)-(e) (2) (i)-(d); (ii)-(a); (iii)-(b); (iv)-(c)

(3) (i)-(e); (ii)-(a); (iii)-(c); (iv)-(d) (4) (i)-(a); (ii)-(c); (iii)-(b); (iv)-(e)

27. An ionic micelle is formed on the addition of

(1) excess water to liquid

Corporate Office : Aakash Tower, 8, Pusa Road, New Delhi-110005 [5]

For More Material Join: @JEEAdvanced_2024


For More Material Join: @JEEAdvanced_2024
Unique Practice Questions Chemistry

(2) excess water to liquid

(3) sodium stearate to pure toluene

(4) liquid diethyl ether to aqueous NaCl solution

28. The number of groups present in a tripeptide Asp – Glu – Lys is _______.

29. Match the following

Type of Colloid Examples

(i) Foam (a) smoke

(ii) Gel (b) cell fluid

(iii) Aerosol (c) jellies

(iv) Emulsion (d) rubber

(e) froth

(f) Milk

(1) (i)-(b), (ii)-(c), (iii)-(e), (iv)-(d) (2) (i)-(d), (ii)-(b), (iii)-(e), (iv)-(f)

(3) (i)-(d), (ii)-(b), (iii)-(a), (iv)-(e) (4) (i)-(e), (ii)-(c), (iii)-(a), (iv)-(f)

30. The intermolecular potential energy for the molecules A, B, C and D given below suggests that :

(1) A-B is the stiffest bond (2) A-D has the shortest bond length

(3) A-A has the largest bond enthalpy (4) D is more electronegative than other atoms

Corporate Office : Aakash Tower, 8, Pusa Road, New Delhi-110005 [6]

For More Material Join: @JEEAdvanced_2024


For More Material Join: @JEEAdvanced_2024
Unique Practice Questions Chemistry

31. The processes of calcination and roasting in metallurgical industries, respectively, can Iead to

(1) Global warming and photochemical smog

(2) Photochemical smog and ozone layer depletion

(3) Photochemical smog and global warming

(4) Global warming and acid rain

32. The reaction in which the hybridisation of the underlined atom is affected is

420 K
(1) XeF4 + SbF5 → (2) H2 SO4 + NaCl ⎯⎯⎯⎯

+
Disproportionation H
(3) H3 PO2 ⎯⎯⎯⎯⎯⎯⎯⎯→ (4) NH3 ⎯⎯⎯

33. The mechanism of action of “Terfenadine” (Seldane) is :

(1) Activates the histamine receptor (2) Helps in the secretion of histamine

(3) Inhibits the secretion of histamine (4) Inhibits the action of histamine receptor

34. The number of chiral centres present in threonine is ________.

35. If a person is suffering from the deficiency of nor-adrenaline, what kind of drug can be suggested?

(1) Analgesic (2) Antidepressant

(3) Anti-inflammatory (4) Antihistamine

36. The number of chiral carbon(s) present in peptide, Ile-Arg-Pro, is _____.

37. The one that is NOT suitable for the removal of permanent hardness of water is

(1) Calgon’s method (2) Ion-exchange method

(3) Clark’s method (4) Treatment with sodium carbonate

38. The incorrect statement is

(1) bronze is an alloy of copper and tin

(2) cast iron is used to manufacture wrought iron

(3) german silver is an alloy of zinc, copper and nickel

(4) brass is an alloy of copper and nickel

Corporate Office : Aakash Tower, 8, Pusa Road, New Delhi-110005 [7]

For More Material Join: @JEEAdvanced_2024


For More Material Join: @JEEAdvanced_2024
Unique Practice Questions Chemistry

39. Consider the following reactions

A → P1 ; B → P2 ; C → P3 ; D → P4,

The order of the above reactions are a, b, c, and d, respectively. The following graph is obtained when log[rate]
vs. log[conc.] are plotted

Among the following, the correct sequence for the order of the reactions is

(1) d>b>a>c (2) d>a>b>c

(3) a>b>c>d (4) c>a>b>d

40. Match the following compounds (Column-I) with their uses (Column-II)

(1) (I)-(B), (II)-(C), (III)-(D), (IV)-(A) (2) (I)-(B), (II)-(D), (III)-(A), (IV)-(C)

(3) (I)-(C), (II)-(D), (III)-(B), (IV)-(A) (4) (I)-(D), (II)-(A), (III)-(C), (IV)-(B)

41. Mischmetal is an alloy consisting mainly of

(1) lanthanoid and actinoid metals (2) actinoid and transition metals

(3) lanthanoid metals (4) actinoid metals

42. For Freundlich adsorption isotherm, a plot of log (x/m) (y-axis) and log p (x-axis) gives a straight line. The
intercept and slope for the line is 0.4771 and 2, respectively. The mass of gas, adsorbed per gram of adsorbent
if the initial pressure is 0.04 atm, is ______ × 10–4 g. (log 3 = 0.4771)

43. The atomic number of Unnilunium is _______.

Corporate Office : Aakash Tower, 8, Pusa Road, New Delhi-110005 [8]

For More Material Join: @JEEAdvanced_2024


For More Material Join: @JEEAdvanced_2024
Unique Practice Questions Chemistry

44. Out of the following, which type of interaction is responsible for the stabilisation of a-helix structure of proteins?

(1) Covalent bonding (2) Hydrogen bonding

(3) Ionic bonding (4) vander Waals forces

45. The major components in “Gun Metal” are:

(1) Cu, Ni and Fe (2) Cu, Sn and Zn

(3) Al, Cu, Mg and Mn (4) Cu, Zn and Ni

46. The gas released during anaerobic degradation of vegetation may lead to :

(1) Acid rain (2) Corrosion of metals

(3) Ozone hole (4) Global warming and cancer

47. Match List-I with List-II.

List-I List-II

(Monomer unit) (Polymer)

(a) Caprolactum (i) Natural rubber

(b) 2-Chloro-1, 3-butadiene (ii) Buna-N

(c) Isoprene (iii) Nylon 6

(d) Acrylonitrile (iv) Neoprene

Choose the correct answer from the options given below:

(1) (a) → (iv), (b) → (iii), (c) → (ii), (d) → (i) (2) (a) → (iii), (b) → (iv), (c) → (i), (d) → (ii)

(3) (a) → (i), (b) → (ii), (c) → (iii), (d) → (iv) (4) (a) → (ii), (b) → (i), (c) → (iv), (d) → (iii)

48. Given below are two statements :

Statement-I : The value of the parameter “Biochemical Oxygen Demand (BOD)” is important for survival of
aquatic life.

Statement-II : The optimum value of BOD is 6.5 ppm.

In the light of the above statements, choose the most appropriate answer from the options given below:

(1) Both statement I and statement II are true (2) Both statement I and statement II are false

(3) Statement I is false but statement II is true (4) Statement I is true but statement II is false

49. Match List-I and List-II.

Corporate Office : Aakash Tower, 8, Pusa Road, New Delhi-110005 [9]

For More Material Join: @JEEAdvanced_2024


For More Material Join: @JEEAdvanced_2024
Unique Practice Questions Chemistry

List-I List-II

(a) Valium (i) Antifertility drug

(b) Morphine (ii) Pernicious anaemia

(c) Norethindrone (iii) Analgesic


(d) Vitamin B12 (iv) Tranquilizer
(1) (a)-(iv), (b)-(iii), (c)-(i), (d)-(ii) (2) (a)-(iv), (b)-(iii), (c)-(ii), (d)-(i)
(3) (a)-(ii), (b)-(iv), (c)-(iii), (d)-(i) (4) (a)-(i), (b)-(iii), (c)-(iv), (d)-(ii)
50. Match List-I with List-II.
List-I (Salt) List-II (Flame colour wavelength)
(a) LiCl (i) 455.5 nm
(b) NaCl (ii) 670.8 nm
(c) RbCl (iii) 780.0 nm
(d) CsCl (iv) 589.2 nm
Choose the correct answer from the options given below:
(1) (a)-(ii), (b)-(i), (c)-(iv), (d)-(iii) (2) (a)-(ii), (b)-(iv), (c)-(iii), (d)-(i)
(3) (a)-(iv), (b)-(ii), (c)-(iii), (d)-(i) (4) (a)-(i), (b)-(iv), (c)-(ii), (d)-(iii)
51. Given below are two statements:
Statement-I : CeO2 can be used for oxidation of aldehyde and ketones.
Statement-II : Aqueous solution of EuSO4 is a strong reducing agent.
In the light of the above statements, choose the correct answer from the options given below:
(1) Both Statement I and Statement II are false (2) Both Statement I and Statement II are true
(3) Statement I is true but Statement II is false (4) Statement I is false but Statement II is true
52. Given below are two statements:
Statement-I : An allotrope of oxygen is an important intermediate in the formation of reducing smog.
Statement-II : Gases such as oxides of nitrogen and sulphur present in troposphere contribute to the fomation
of photochemical smog.
In the light of the above statements, choose the correct answer from the options given below:
(1) Statement I is true but Statement II is false (2) Both Statement I and Statement II are false
(3) Statement I is false but Statement II is true (4) Both Statement I and Statement II are true
53. Compound(s) which will liberate carbon dioxide with sodium bicarbonate solution is/are :

Corporate Office : Aakash Tower, 8, Pusa Road, New Delhi-110005 [10]

For More Material Join: @JEEAdvanced_2024


For More Material Join: @JEEAdvanced_2024
Unique Practice Questions Chemistry

(1) B only (2) C only

(3) A and B only (4) B and C only

54. The major components of German Silver are:

(1) Cu, Zn and Ni (2) Zn, Ni and Ag

(3) Ge, Cu and Ag (4) Cu, Zn and Ag

55. The correct sequence of reagents used in the preparation of 4-bromo-2-nitroethyl benzene from benzene is:

(1) CH3COCl/AlCl3, Zn-Hg/HCl, Br2/AlBr3, HNO3/H2SO4

(2) HNO3/H2SO4, Br2/AlCl3, CH3COCl/AlCl3, Zn-Hg/HCl

(3) CH3COCl/AlCl3, Br2/AlBr3, HNO3/H2SO4, Zn/HCl

(4) Br2/AlBr3, CH3COCl/AlCl3, HNO3/H2SO4, Zn/HCl

56. Match List-I with List-II.

List-I (Ore) List-II (Element Present)

(a) Kernite (i) Tin

(b) Cassiterite (ii) Boron

(c) Calamine (iii) Fluorine

(d) Cryolite (iv) Zinc

Choose the most appropriate answer from the options given below:

(1) (a)(ii), (b)(i), (c)(iv), d(iii) (2) (a)(iii), (b)(i), (c)(ii), d(iv)

Corporate Office : Aakash Tower, 8, Pusa Road, New Delhi-110005 [11]

For More Material Join: @JEEAdvanced_2024


For More Material Join: @JEEAdvanced_2024
Unique Practice Questions Chemistry

(3) (a)(ii), (b)(iv), (c)(i), d(iii) (4) (a)(i), (b)(iii), (c)(iv), d(ii)

57. The presence of ozone in troposphere :

(1) Protects us from greenhouse effect (2) Protects us from the X-ray radiation

(3) Generates photochemical smog (4) Protects us from the UV radiation

58. Which of the following vitamin is helpful in delaying the blood clotting?

(1) Vitamin B (2) Vitamin E

(3) Vitamin K (4) Vitamin C

59. Calgon is used for water treatment. Which of the following statement is NOT true about Calgon?

(1) It is polymeric compound and is water soluble

(2) Calgon contains the 2nd most abundant element by weight in the Earth’s crust

(3) It is also known as Graham’s salt

(4) It doesnot remove Ca2+ ion by precipitation

60. Match List-I with List-II.

List-I List-II

(a) Sucrose (i) -D-Galactose and -D-Glucose

(b) Lactose (ii) -D-Glucose and -D-Fructose

(c) Maltose (iii) -D-Glucose and -D-Glucose

Choose the correct answer from the options given below :

(1) (a) → (i), (b) → (iii), (c) → (ii) (2) (a) → (iii), (b) → (ii), (c) → (i)

(3) (a) → (ii), (b) → (i), (c) → (iii) (4) (a) → (iii), (b) → (i), (c) → (ii)

61. Which among the following pairs of Vitamins is stored in our body relatively for longer duration?

(1) Ascorbic acid and Vitamin D (2) Vitamin A and Vitamin D

(3) Thiamine and Ascorbic acid (4) Thiamine and Vitamin A

62. Match List-I with List-II :

List-I (Industrial process) List-II (Application)

Corporate Office : Aakash Tower, 8, Pusa Road, New Delhi-110005 [12]

For More Material Join: @JEEAdvanced_2024


For More Material Join: @JEEAdvanced_2024
Unique Practice Questions Chemistry

(a) Haber’s process (i) HNO3 synthesis

(b) Ostwald’s process (ii) Aluminium extraction

(c) Contact process (iii) NH3 synthesis

(d) Hall-Heroult process (iv) H2SO4 synthesis

Choose the correct answer from the options given below.

(1) (a)-(ii), (b)-(iii), (c)-(iv), (d)-(i) (2) (a)-(iii), (b)-(iv), (c)-(i), (d)-(ii)

(3) (a)-(iv), (b)-(i), (c)-(ii), (d)-(iii) (4) (a)-(iii), (b)-(i), (c)-(iv), (d)-(ii)

63. The type of pollution that gets increased during the day time and in the presence of O 3 is

(1) Reducing smog (2) Acid rain

(3) Global warming (4) Oxidising smog

64. Given below are two statement : one is labelled as Assertion A and the other is labelled as Reason R :

Assertion A : Size of Bk3+ ion is less than Np3+ ion.

Reason R : The above is a consequence of the lanthanoid contraction.

In the light of the above statements, choose the correct answer from the options given below.

(1) A is false but R is true

(2) A is true but R is false

(3) Both A and R are true but R is not the correct explanation of A

(4) Both A and R are true and R is the correct explanation of A

65. The INCORRECT statements below regarding colloidal solutions is

(1) The flocculating power of Al3+ is more than that of Na+.

(2) A colloidal solution shows Brownian motion of colloidal particles.

(3) An ordinary filter paper can stop the flow of colloidal particles.

(4) A colloidal solution shows colligative properties.

66. Match List-I with List-II :

List-I List-II

Test/Reagents/ Species detected

Corporate Office : Aakash Tower, 8, Pusa Road, New Delhi-110005 [13]

For More Material Join: @JEEAdvanced_2024


For More Material Join: @JEEAdvanced_2024
Unique Practice Questions Chemistry

Observation(s)

(a) Lassaigne’s Test (i) Carbon

(b) Cu(II) oxide (ii) Sulphur

(c) Silver nitrate (iii) N, S, P, and halogen

(d) The sodium fusion (iv) Halogen Specifically

extract gives black

precipitate with acetic

acid and lead acetate

The correct match is:

(1) (a)-(iii), (b)-(i), (c)-(iv), (d)-(ii)

(2) (a)-(i), (b)-(iv), (c)-(iii), (d)-(ii)

(3) (a)-(iii), (b)-(i), (c)-(ii), (d)-(iv)

(4) (a)-(i), (b)-(ii), (c)-(iv), (d)-(iii)

67. The green house gas/es is (are):

(A) Carbon dioxide (B) Oxygen

(C) Water vapour (D) Methane

Choose the most appropriate answer from the options given below:

(1) (A), (C) and (D) only (2) (A) and (B) only

(3) (A) and (C) only (4) (A) only

68. A colloidal system consisting of a gas dispersed in a solid is called a/an

(1) aerosol (2) foam

(3) solid sol (4) gel

69. Given below are two statements :

Statement-I : Retardation factor (Rf) can be measured in meter/centimeter.

Statement-II : Rf value of a compound remains constant in all solvents.

Choose the most appropriate answer from the options given below :

(1) Statement I is true but statement II is false (2) Both statement I and statement II are true

Corporate Office : Aakash Tower, 8, Pusa Road, New Delhi-110005 [14]

For More Material Join: @JEEAdvanced_2024


For More Material Join: @JEEAdvanced_2024
Unique Practice Questions Chemistry

(3) Both statement I and statement II are false (4) Statement I is false but statement II is true

70. The INCORRECT statement(s) about heavy water is (are)

(A) used as a moderator in nuclear reactor

(B) obtained as a by-product in fertilizer industry

(C) used for the study of reaction mechanism

(D) has a higher dielectric constant than water

Choose the correct answer from the options given below:

(1) (B) and (D) only (2) (B) only

(3) (D) only (4) (C) only

71. With respect to drug-enzyme interaction, identify the wrong statement.

(1) Allosteric inhibitor competes with the enzyme’s active site

(2) Allosteric inhibitor changes the enzyme’s active site

(3) Non-Competitive inhibitor binds to the allosteric site

(4) Competitive inhibitor binds to the enzyme’s active site

Enzyme A
72. C12H22O11 + H2O ⎯⎯⎯⎯⎯
→ C6H12O6 + C6H12O6
Glucose Fructose

Enzyme B
C6H12O6 ⎯⎯⎯⎯⎯
→ 2C2H5OH + 2CO2
Glucose

In the above reactions, the enzyme A and enzyme B respectively are

(1) Amylase and Invertase (2) Invertase and Zymase

(3) Zymase and Invertase (4) Invertase and Amylase

73. Match List-I with List-II.

List-I (Chemical Compound) List-II (Used as)

(a) Sucralose (i) Synthetic detergent

(b) Glyceryl ester (ii) Artificial sweetener

of stearic acid

Corporate Office : Aakash Tower, 8, Pusa Road, New Delhi-110005 [15]

For More Material Join: @JEEAdvanced_2024


For More Material Join: @JEEAdvanced_2024
Unique Practice Questions Chemistry

(c) Sodium benzoate (iii) Antiseptic

(d) Bithionol (iv) Food preservative

Choose the correct match.

(1) (a)-(i), (b)-(ii), (c)-(iv), (d)-(iii) (2) (a)-(iv), (b)-(iii), (c)-(ii), (d)-(i)

(3) (a)-(ii), (b)-(i), (c)-(iv), (d)-(iii) (4) (a)-(iii), (b)-(ii), (c)-(iv), (d)-(i)

74. Which of the following statement(s) is(are) incorrect reason for eutrophication?

(A) Excess usage of fertilisers

(B) Excess usage of detergents

(C) Dense plant population in water bodies

(D) Lack of nutrients in water bodies that prevent plant growth

Choose the most appropriate answer from the options given below.

(1) (D) Only (2) (A) only

(3) (B) and (D) only (4) (C) only

75. Match List-I with List-II :

List-I List-II

(a) Ca(OCl)2 (i) Antacid

1
(b) CaSO4 · H2O (ii) Cement
2

(c) CaO (iii) Bleach

(d) CaCO3 (iv) Plaster of Paris

Choose the most appropriate answer from the options given below :

(1) (a)-(iii), (b)-(ii), (c)-(i), (d)-(iv) (2) (a)-(iii), (b)-(ii), (c)-(iv), (d)-(i)

(3) (a)-(iii), (b)-(iv), (c)-(ii), (d)-(i) (4) (a)-(i), (b)-(iv), (c)-(iii), (d)-(ii)

76. Match List-I with list-II :

List-I List-II

(a) Chlorophyll (i) Ruthenium

(b) Vitamin - B12 (ii) Platinum

Corporate Office : Aakash Tower, 8, Pusa Road, New Delhi-110005 [16]

For More Material Join: @JEEAdvanced_2024


For More Material Join: @JEEAdvanced_2024
Unique Practice Questions Chemistry

(c) Anticancer drug (iii) Cobalt

(d) Grubbs catalyst (iv) Magnesium

Choose the most appropriate answer from the options given below :

(1) (a)-(iv), (b)-(iii), (c)-(ii), (d)-(i) (2) (a)-(iv), (b)-(ii), (c)-(iii), (d)-(i)

(3) (a)-(iii), (b)-(ii), (c)-(iv), (d)-(i) (4) (a)-(iv), (b)-(iii), (c)-(i), (d)-(ii)

77. Match List-I with list-II :

List-I (Class of Drug) List-II (Example)

(a) Antacid (i) Novestrol

(b) Artificial Sweetener (ii) Cimetidine

(c) Antifertility (iii) Valium

(d) Tranquilizers (iv) Alitame

Choose the most appropriate match :

(1) (a)-(iv), (b)-(iii), (c)-(i), (d)-(ii) (2) (a)-(ii), (b)-(iv), (c)-(i), (d)-(iii)

(3) (a)-(iv), (b)-(i), (c)-(ii), (d)-(iii) (4) (a)-(ii), (b)-(iv), (c)-(iii), (d)-(i)

78. Match List-I and with List-II.

List-I (Process) List-II (Catalyst)

(a) Deacon's process (i) ZSM-5

(b) Contact process (ii) CuCl2

(c) Cracking of (iii) Particles 'Ni'

hydrocarbons

(d) Hydrogenation of (iv) V2O5

vegetable oils

Choose the most appropriate answer from the options given below :

(1) (a)-(iv), (b)-(ii), (c)-(i), (d)-(iii) (2) (a)-(ii), (b)-(iv), (c)-(i), (d)-(iii)

(3) (a)-(iii), (b)-(i), (c)-(iv), (d)-(ii) (4) (a)-(i), (b)-(iii), (c)-(ii), (d)-(iv)

79. The chemical that is added to reduce the melting point of the reaction mixture during the extraction of
aluminium is

Corporate Office : Aakash Tower, 8, Pusa Road, New Delhi-110005 [17]

For More Material Join: @JEEAdvanced_2024


For More Material Join: @JEEAdvanced_2024
Unique Practice Questions Chemistry

(1) Cryolite (2) Calamine

(3) Kaolite (4) Bauxite

80. Given below are two statements :

Statement I : Non-biodegradable wastes are generated by the thermal power plants.

Statement II : Bio-degradable detergents leads to eutrophication.

In the light of the above statements, choose the most appropriate answer from the options given below :
(1) Both statement I and statement II are false (2) Statement I is true but statement II is false
(3) Statement I is false but statement II is true (4) Both statement I and statement II are true
81. Deficiency of vitamin K causes :
(1) Cheilosis (2) Increase in blood clotting time
(3) Increase in fragility of RBC’s (4) Decrease in blood clotting time
82. Match List-I with List-II :
List-I (Class of Chemicals) List-II (Example)
(a) Antifertility drug (i) Meprobamate
(b) Antibiotic (ii) Alitame
(c) Tranquilizer (iii) Norethindrone
(d) Artificial Sweetener (iv) Salvarsan
Choose the most appropriate match :
(1) (a)-(iii), (b)-(iv), (c)-(i), (d)-(ii) (2) (a)-(iv), (b)-(iii), (c)-(ii), (d)-(i)
(3) (a)-(ii), (b)-(iii), (c)-(iv), (d)-(i) (4) (a)-(ii), (b)-(iv), (c)-(i), (d)-(iii)
83. Match List-I with List-II :
List-I List-II
(a) Mercury (i) Vapour phase refining
(b) Copper (ii) Distillation refining
(c) Silicon (iii) Electrolytic refining
(d) Nickel (iv) Zone refining
Choose the most appropriate answer from the option given below.
(1) (a)-(ii), (b)-(iv), (c)-(iii), (d)-(i) (2) (a)-(ii), (b)-(iii), (c)-(i), (d)-(iv)
(3) (a)-(i), (b)-(iv), (c)-(ii), (d)-(iii) (4) (a)-(ii), (b)-(iii), (c)-(iv), (d)-(i)
84. Match List-I with List-II :
List-I List-II
(a) Be (i) treatment of cancer
(b) Mg (ii) extraction of metals

Corporate Office : Aakash Tower, 8, Pusa Road, New Delhi-110005 [18]

For More Material Join: @JEEAdvanced_2024


For More Material Join: @JEEAdvanced_2024
Unique Practice Questions Chemistry

(c) Ca (iii) incendiary bombs and signals


(d) Ra (iv) windows of X-ray tubes
(v) bearings for motor engines
Choose the most appropriate answer from the option given below.

(1) (a)-(iii), (b)-(iv), (c)-(ii), (d)-(v) (2) (a)-(iv), (b)-(iii), (c)-(i), (d)-(ii)

(3) (a)-(iii), (b)-(iv), (c)-(v), (d)-(ii) (4) (a)-(iv), (b)-(iii), (c)-(ii), (d)-(i)

85. The conditions given below are in the context of observing Tyndall effect in colloidal solutions:

(A) The diameter of the colloidal particles is comparable to the wavelength of light used.

(B) The diameter of the colloidal particles is much smaller than the wavelength of light used.

(C) The diameter of the colloidal particles is much larger than the wavelength of light used.

(D) The refractive indices of the dispersed phase and the dispersion medium are comparable.

(E) The dispersed phase has a very different refractive index from the dispersion medium.

Choose the most appropriate conditions from the options given below.

(1) (A) and (D) only (2) (C) and (D) only

(3) (B) and (E) only (4) (A) and (E) only

86. An inorganic Compound ‘X’ on treatment with concentrated H 2SO4 produces brown fumes and gives dark
brown ring with FeSO4 in presence of concentrated H2SO4. Also Compound ‘X’ gives precipitate ‘Y’, when its
solution in dilute HCl is treated with H2S gas. The precipitate ‘Y’ on treatment with concentrated HNO3 followed
by excess of NH4OH further gives deep blue coloured solution, Compound ‘X’ is

(1) Cu(NO3)2 (2) Pb(NO2)2

(3) Pb(NO3)2 (4) Co(NO3)2

87. Identify the incorrect statement from the following

(1) Amylose is a branched chain polymer of glucose

(2) -Glycosidic linkage makes cellulose polymer

(3) Glycogen is called as animal starch

(4) Starch is a polymer of -D glucose

88. Which one of the following statements is not true about enzymes ?

(1) The action of enzymes is temperature and pH specific

Corporate Office : Aakash Tower, 8, Pusa Road, New Delhi-110005 [19]

For More Material Join: @JEEAdvanced_2024


For More Material Join: @JEEAdvanced_2024
Unique Practice Questions Chemistry

(2) Enzymes are non-specific for a reaction and substrate

(3) Enzymes work as catalysts by lowering the activation energy of a biochemical reaction

(4) Almost all enzymes are proteins

89. Two radioactive substances A and B have decay constants 5 and  respectively. At t = 0, a sample has the
2
 1
same number of the two nuclei. The time taken for the ratio of the number of nuclei to become   will be :
 e

1 1
(1) (2)
2 4
1 2
(3) (4)
 

90. Thiamine and pyridoxine are also known respectively as:

(1) Vitamin E and Vitamin B2 (2) Vitamin B2 and Vitamin E

(3) Vitamin B1 and Vitamin B6 (4) Vitamin B6 and Vitamin B2

91. Match List-I and List-II:

List-I (Elements) List-II (Properties)

(a) Ba (i) Organic solvent soluble compounds

(b) Ca (ii) Outer electronic configuration 6s2

(c) Li (iii) Oxalate insoluble in water

(d) Na (iv) Formation of very strong monoacidic base

Choose the correct answer from the options given below:

(1) (a)-(iii), (b)-(ii), (c)-(iv) and (d)-(i) (2) (a)-(ii), (b)-(iii), (c)-(i) and (d)-(iv)

(3) (a)-(i), (b)-(iv), (c)-(ii) and (d)-(iii) (4) (a)-(iv), (b)-(i), (c)-(ii) and (d)-(iii)

92. The idea of froth floatation method came from a person X and this method is related to the process Y of ores.
X and Y, respectively, are :
(1) Fisher woman and concentration (2) Washer woman and concentration
(3) Washer man and reduction (4) Fisher man and reduction

93. Which one of the following compounds of Groups-14 elements is not known?

(1) [GeCl6]2– (2) [SiF6]2–

(3) [Sn(OH)6]2– (4) [SiCl6]2–

Corporate Office : Aakash Tower, 8, Pusa Road, New Delhi-110005 [20]

For More Material Join: @JEEAdvanced_2024


For More Material Join: @JEEAdvanced_2024
Unique Practice Questions Chemistry

94. Sodium stearate CH3(CH2)16COO–Na+ is an anionic surfactant which forms micelles in water. Choose the
correct statement for it from the following

(1) It forms non-spherical micelles with CH3(CH2)16 – group pointing towards the centre

(2) It forms spherical micelles with CH3(CH2)16 – group pointing outwards on the surface of sphere

(3) It forms non-spherical micelles with − COO− group pointing outwards on the surface

(4) It forms spherical micelles with CH3(CH2)16 – group pointing towards the centre of sphere

95. is a repeating unit for

(1) Acrilan (2) Novolac

(3) Buna-N (4) Neoprene

96. The water soluble protein is

(1) Albumin (2) Collagen

(3) Myosin (4) Fibrin

97. Given below are two statements :

Statement I : Chlorofluoro carbons breakdown by radiation in the visible energy region and release chlorine
gas in the atmosphere which then reacts with stratospheric ozone.

Statement II : Atmospheric ozone reacts with nitric oxide to give nitrogen and oxygen gases, which add to the
atmosphere.

For the above statements choose the correct answer from the options given below :

(1) Statement I is incorrect but statement II is true

(2) Statement I is correct but statement II is false

(3) Both statement I and II are correct

(4) Both statement I and II are false

98. Match List-I with List-II :

List-I List-II

Corporate Office : Aakash Tower, 8, Pusa Road, New Delhi-110005 [21]

For More Material Join: @JEEAdvanced_2024


For More Material Join: @JEEAdvanced_2024
Unique Practice Questions Chemistry

Example Classification

of Colloids

(a) Cheese (i) Dispersion of liquid in liquid

(b) Pumice stone (ii) Dispersion of liquid in gas

(c) Hair cream (iii) Dispersion of gas in solid

(d) Cloud (iv) Dispersion of liquid in solid

Choose the most appropriate answer from the options given below :

(1) (a) - (iv), (b) - (iii), (c) - (i), (d) - (ii) (2) (a) - (iv), (b) - (i), (c) - (iii), (d) - (ii)

(3) (a) - (iii), (b) - (iv), (c) - (i), (d) - (ii) (4) (a) - (iv), (b) - (iii), (c) - (ii), (d) - (i)

99. Match List-I with List-II : (Both having metallurgical terms)

List-I List-II

(a) Concentration of Ag ore (i) Reverberatory furnace

(b) Blast furnace (ii) Pig iron

(c) Blister copper (iii) Leaching with dilute NaCN solution

(d) Froth floatation method (iv) Sulfide ores

Choose the correct answer from the options given below :

(1) (a) - (iv), (b) - (iii), (c) - (ii), (d) - (i) (2) (a) - (iii), (b) - (iv), (c) - (i), (d) - (ii)

(3) (a) - (iii), (b) - (ii), (c) - (i), (d) - (iv) (4) (a) - (iv), (b) - (i), (c) - (iii), (d) - (ii)

100. A biodegradable polyamide can be made from :

(1) Glycine and isoprene (2) Glycine and aminocaproic acid

(3) Styrene and caproic acid (4) Hexamethylene diamine and adipic acid

101. Which one of the following metals forms interstitial hydride easily?

(1) Mn (2) Cr

(3) Fe (4) Co

102. Match List-I with List-II

List-I List-II

(a) NaOH (i) Acidic

Corporate Office : Aakash Tower, 8, Pusa Road, New Delhi-110005 [22]

For More Material Join: @JEEAdvanced_2024


For More Material Join: @JEEAdvanced_2024
Unique Practice Questions Chemistry

(b) Be(OH)2 (ii) Basic

(c) Ca(OH)2 (iii) Amphoteric

(d) B(OH)3

(e) Al(OH)3

Choose the most appropriate answer from the options given below :

(1) (a)-(ii), (b)-(ii), (c)-(iii), (d)-(i), (e)-(iii) (2) (a)-(ii), (b)-(ii), (c)-(iii), (d)-(ii), (e)-(iii)

(3) (a)-(ii), (b)-(i), (c)-(ii), (d)-(iii), (e)-(iii) (4) (a)-(ii), (b)-(iii), (c)-(ii), (d)-(i), (e)-(iii)

103.

The compound ‘A’ is a complementary base of ___in DNA strands.


(1) Uracil (2) Guanine
(3) Adenine (4) Cytosine
104. Which one of the following statements is NOT correct?
(1) Eutrophication leads to increase in the oxygen level in water
(2) Eutrophication indicates that water body is polluted
(3) Eutrophication leads to anaerobic conditions
(4) The dissolved oxygen concentration below 6 ppm inhibits fish growth
105. Which one among the following chemical tests is used to distinguish monosaccharide from disaccharide?
(1) Seliwanoff’s test (2) Iodine test
(3) Tollen’s test (4) Barfoed test
106. Match List-I with List-II
List-I (Drug) List-II (Class of Drug)
(a) Furacin (i) Antibiotic
(b) Arsphenamine (ii) Tranquilizers
(c) Dimetone (iii) Antiseptic
(d) Valium (iv) Synthetic antihistamines
Choose the most appropriate match
(1) (a)-(iii), (b)-(iv), (c)-(ii), (d)-(i) (2) (a)-(ii), (b)-(i), (c)-(iii), (d)-(iv)
(3) (a)-(i), (b)-(iii), (c)-(iv), (d)-(ii) (4) (a)-(iii), (b)-(i), (c)-(iv), (d)-(ii)
107. Given below are two statements :

Corporate Office : Aakash Tower, 8, Pusa Road, New Delhi-110005 [23]

For More Material Join: @JEEAdvanced_2024


For More Material Join: @JEEAdvanced_2024
Unique Practice Questions Chemistry

Statement I : Penicillin is a bacteriostatic type antibiotic.

Statement II : The general structure of Penicillin is

Choose the correct option :


(1) Statement I is incorrect but Statement II is true (2) Statement I is correct but Statement II is false
(3) Both Statement I and Statement II are true (4) Both Statement I and Statement II are false
108. The addition of silica during the extraction of copper from its sulphide ore
(1) Converts iron oxide into iron silicate
(2) Converts copper sulphide into copper silicate
(3) Reduces copper sulphide into metallic copper
(4) Reduces the melting point of the reaction mixture
109. Match List-I with List-II
List-I (compound) List-II (effect/affected species)
(a) Carbon monoxide (i) Carcinogenic
(b) Sulphur dioxide (ii) Metabolized by pyrus plants
(c) Polychlorinated biphenyls (iii) Haemoglobin
(d) Oxides of nitrogen (iv) Stiffness of flower buds
Choose the correct answer from the options given below :
(1) (a) - (i), (b) - (ii), (c) - (iii), (d) - (iv) (2) (a) - (iii), (b) - (iv), (c) - (i), (d) - (ii)
(3) (a) - (iv), (b) - (i), (c) - (iii), (d) - (ii) (4) (a) - (iii), (b) - (iv), (c) - (ii), (d) - (i)
110. Which one of the following methods is most suitable for preparing deionized water?
(1) Synthetic resin method (2) Permutit method
(3) Clark’s method (4) Calgon’s method
111. The total number of negative charge in the tetrapeptide, Gly-Glu-Asp-Tyr, at pH 12.5 will be ______. (Integer
answer)

112.

Corporate Office : Aakash Tower, 8, Pusa Road, New Delhi-110005 [24]

For More Material Join: @JEEAdvanced_2024


For More Material Join: @JEEAdvanced_2024
Unique Practice Questions Chemistry

The class of drug to which chlordiazepoxide with above structure belongs is :


(1) Antacid (2) Analgesic
(3) Tranquilizer (4) Antibiotic
113. Given below are two statements : one is labelled as Assertion (A) and the other is labelled as Reason (R).

Assertion (A) : Sucrose is a disaccharide and a non-reducing sugar.

Reason (R) : Sucrose involves glycosidic linkage between C1 of -glucose and C2 of -fructose.

the most appropriate answer from the options given below :

(1) Both (A) and (R) are true and (R) is the true explanation of (A)

(2) (A) is true but (R) is false

(3) Both (A) and (R) are true but (R) is not the true explanation of (A).

(4) (A) is false but (R) is true

114. Given below are two statements : one is labelled as Assertion (A) and the other is labelled as Reason (R)

Assertion (A) : Photochemical smog causes cracking of rubber.

Reason (R) : Presence of ozone, nitric oxide,


acrolein, formaldehyde and peroxyacetyl nitrate in photochemical smog makes it oxidizing

Choose the most appropriate answer from the options given below:

(1) Both (A) and (R) are true and (R) is the true explanation of (A)

(2) (A) is true but (R) is false

(3) (A) is false but (R) is true

(4) Both (A) and (R) are true but (R) is not the true explanation of (A).

115. Match items of List - I with those of List - II :

List – I (Property) List – II (Example)

(a) Diamagnetism (i) MnO

(b) Ferrimagnetism (ii) O2

(c) Paramagnetism (iii) NaCl

(d) Antiferromagnetism (iv) Fe3O4

Choose the most appropriate answer from the options given below :

Corporate Office : Aakash Tower, 8, Pusa Road, New Delhi-110005 [25]

For More Material Join: @JEEAdvanced_2024


For More Material Join: @JEEAdvanced_2024
Unique Practice Questions Chemistry

(1) (a)-(iv), (b)-(ii), (c)-(i), (d)-(iii) (2) (a)-(i), (b)-(iii), (c)-(iv), (d)-(ii)

(3) (a)-(iii), (b)-(iv), (c)-(ii), (d)-(i) (4) (a)-(ii), (b)-(i), (c)-(iii), (d)-(iv)

116.

The correct statement about (A), (B), (C) and (D) is :


(1) (B), (C) and (D) are tranquilizers (2) (B) and (C) are tranquilizers
(3) (A) and (D) are tranquilizers (4) (A), (B) and (C) are narcotic analgesics
117. Which one of the following tests used for the identification of functional groups in organic compounds does not
use copper based reagent?
(1) Seliwanoff’s test (2) Barfoed’s test
(3) Benedict’s test (4) Biuret test for peptide bond
118. Lyophilic sols are more stable than lyophobic sols because,
(1) The colloidal particles are solvated
(2) The colloidal particles have positive charge
(3) The colloidal particles have no charge
(4) There is a strong electrostatic repulsion between the negatively charged colloidal particles
119. The denticity of an organic ligand, biuret is :
(1) 3 (2) 2
(3) 4 (4) 6
120. Given below are two statements :
Statement I : The process of producing syn-gas is called gasification of coal.
Statement II : The composition of syn-gas is CO + CO2 + H2 (1 : 1 : 1).
In the light of the above statements, choose the most appropriate answer from the options given below :
(1) Statement I is true but Statement II is false (2) Both Statement I and Statement II are true
(3) Both Statement I and Statement II are false (4) Statement I is false but Statement II is true

Corporate Office : Aakash Tower, 8, Pusa Road, New Delhi-110005 [26]

For More Material Join: @JEEAdvanced_2024


For More Material Join: @JEEAdvanced_2024
Unique Practice Questions Chemistry

121. Which of the following is NOT an example of fibrous protein?


(1) Albumin (2) Collagen
(3) Myosin (4) Keratin
122. The deposition of X and Y on ground surfaces is referred as wet and dry depositions, respectively. X and Y
are

(1) X = SO2, Y = Ammonium salts (2) X = CO2, Y = SO2

(3) X = Ammonium salts, Y = SO2 (4) X = Ammonium salts, Y = CO2

123. Water sample is called cleanest on the basis of which one of the BOD values given below :

(1) 3 ppm (2) 21 ppm

(3) 15 ppm (4) 11 ppm

124. Match List-I with List-II.


List-I (Colloid Preparation Method) List-II (Chemical Reaction)
(a) Hydrolysis (i) 2AuCl3 + 3HCHO + 3H2O → 2Au(sol) + 3HCOOH +
6HCl
(b) Reduction (ii) As2O3 + 3H2S → As2S3(sol) + 3H2O
(c) Oxidation (iii) SO2 + 2H2S → 3S(sol) + 2H2O
(d) Double Decomposition (iv) FeCl3 + 3H2O → Fe(OH)3(sol) + 3HCl
Choose the most appropriate answer from the options given below
(1) (a)-(iv), (b)-(ii), (c)-(iii), (d)-(i) (2) (a)-(i), (b)-(ii), (c)-(iv), (d)-(iii)
(3) (a)-(i), (b)-(iii), (c)-(ii), (d)-(iv) (4) (a)-(iv), (b)-(i), (c)-(iii), (d)-(ii)
125. The correct match between Item-I and Item-II is
Item-I (drug) Item-II (test)
A. Chloroxylenol P. Carbylamine test
B. Norethindrone Q. Sodium hydrogen Carbonate test
C. Sulphapyridine R. Ferric chloride test
D. Penicillin S. Bayer's test
(1) A → Q, B → P, C → S, D → R (2) A → R, B → S, C → P, D → Q
(3) A → Q, B → S, C → P, D → R (4) A → R, B → P, C → S, D → Q
126. The alkaline earth metal nitrate that does not crystallise with water molecules, is
(1) Ba(NO3)2 (2) Ca(NO3)2
(3) Mg(NO3)2 (4) Sr(NO3)2

127.
The correct match between Item-I and Item-II is :
Item-I Item-II
(A) Benzaldehyde (P) Mobile phase
(B) Alumina (Q) Adsorbent

Corporate Office : Aakash Tower, 8, Pusa Road, New Delhi-110005 [27]

For More Material Join: @JEEAdvanced_2024


For More Material Join: @JEEAdvanced_2024
Unique Practice Questions Chemistry

(C) Acetonitrile (R) Adsorbate


(1) (A) → (Q), (B) → (R), (C) → (P) (2) (A) → (Q), (B) → (P), (C) → (R)
(3) (A) → (P), (B) → (R), (C) → (Q) (4) (A) → (R), (B) → (Q), (C) → (P)
128. The correct sequnce of amino acids present in the tripeptide given below is :
The given tripeptide contains.

(1) Leu - Ser - Thr (2) Thr - Ser - Val


(3) Val - Ser - Thr (4) Thr - Ser – Leu
129. If dichloromethane (DCM) and water (H2O) are used for differential extraction, which one of the following

statements is correct?
(1) DCM and H2O will make turbid/colloidal mixture

(2) DCM and H2O will be miscible clearly

(3) DCM and H2O would stay as upper and lower layer respectively in the separating funnel (S.F.)

(4) DCM and H2O would stay as lower and upper layer respectively in the S.F.

130. The correct structure of product ‘P’ in the following reaction is


NEt
Asn − Ser + (CH3CO)2 O ⎯⎯⎯
3
→P
(excess)

(1) (2)

(3) (4)

131. The correct match between item ‘I’ and item ‘II’ is :
Item ‘I’ Item ‘II’
(compound) (reagent)
(A) Lysine (P) 1-naphthol

Corporate Office : Aakash Tower, 8, Pusa Road, New Delhi-110005 [28]

For More Material Join: @JEEAdvanced_2024


For More Material Join: @JEEAdvanced_2024
Unique Practice Questions Chemistry

(B) Furfural (Q) ninhydrin


(C) Benzyl alcohol (R) KMnO4

(D) Styrene (S) Ceric ammonium nitrate


(1) (A) → (R); (B) → (P); (C) → (Q); (D) → (S) (2) (A) → (Q); (B) → (P); (C) → (S); (D) → (R)
(3) (A) → (Q); (B) → (R); (C) → (S); (D) → (P) (4) (A) → (Q); (B) → (P); (C) → (R); (D) → (S)
132. The major product obtained in the following reaction is:

(1) (2)

(3) (4)

133. The major porduct of the following reaction is :

(1) (2)

(3) (4)

134. Match the metals (column I) with the coordination compound(s)/enzyme(s) (column II)
(Column I) (Column II)
Metals Coordination compound(s)/ enzyme(s)
(A) Co (i) Wilkinson catalyst
(B) Zn (ii) Chlorophyll
(C) Rh (iii) Vitamin B12
(D) Mg (iv) Carbonic anhydrase
(1) (A) - (iv), (B) - (iii), (C) - (i), (D) - (ii) (2) (A) - (i), (B) - (ii), (C) - (iii), (D) - (iv)
(3) (A) - (ii), (B) - (i), (C) - (iv), (D) - (iii) (4) (A) - (iii), (B) - (iv), (C) - (i), (D) - (ii)
135. The correct match between Item-I and Item-II is:
Item-I Item-II
(A) Ester test (P) Tyr
(B) Carbylamine test (Q) AsP
(C) Phthalein dye test (R) Ser

Corporate Office : Aakash Tower, 8, Pusa Road, New Delhi-110005 [29]

For More Material Join: @JEEAdvanced_2024


For More Material Join: @JEEAdvanced_2024
Unique Practice Questions Chemistry

(S) Lys
(1) (A) → (Q); (B) → (S); (C) → (P) (2) (A) → (R); (B) → (Q); (C) → (P)
(3) (A) → (Q); (B) → (S); (C) → (R) (4) (A) → (R); (B) → (S); (C) → (Q)
136. The correct match between Item-I and Item-II is:
Item-I Item-II
(A) Allosteric effect (P) Molecule binding to the active site of enzyme
(B) Competitive (Q) Molecule crucial for inhibitor communication in the
body
(C) Receptor (R) Molecule binding to a site other than the active site
of enzyme
(D) Poison (S) Molecule binding to the enzyme covalently
(1) (A) → (P); (B) → (R); (C) → (S); (D) → (Q) (2) (A) → (R); (B) → (P); (C) → (Q); (D) → (S)
(3) (A) → (P); (B) → (R); (C) → (Q); (D) → (S) (4) (A) → (R); (B) → (P); (C) → (S); (D) → (Q)
137. Which of the following compounds reacts with ethylmagnesium bromide and also decolourizes bromine water
solution?

(1) (2)

(3) (4)

138. The standard electrode potential and its temperature coefficientfor a cell are 2 V and – 5 × 10–4 VK–1

at 300 K respectively. The cell reaction is


Zn(s) + Cu2+(aq) → Zn2+ (aq) + Cu(s)
The standard reaction enthalpy (r) at 300 K in kJ mol–1 is,

[Use R = 8 JK–1 mol–1 and F = 96,000 C mol–1]


(1) 206.4 (2) –384.0
(3) –412.8 (4) 192.0
139. The correct statement(s) among I to III with respect to potassium ions that are abundant within the cell fluids
is/are
I. They activate many enzymes
II. They participate in the oxidation of glucose to produce ATP
III. Along with sodium ions, they are responsible for the transmission of nerve signals

Corporate Office : Aakash Tower, 8, Pusa Road, New Delhi-110005 [30]

For More Material Join: @JEEAdvanced_2024


For More Material Join: @JEEAdvanced_2024
Unique Practice Questions Chemistry

(1) I and III only (2) I, II and III


(3) III only (4) I and II only
140. The correct structure of histidine in a strongly acidic solution (pH = 2) is

(1) (2)

(3) (4)

141. The major product of the following reaction is

(1) (2)

(3) (4)

142. Polysubstitution is a major drawback in :


(1) Reimer Tiemann reaction (2) Acetylation of aniline
(3) Friedel Craft’s acylation (4) Friedel Craft’s alkylation
143. Fructose and glucose can be distinguished by:
(1) Fehling’s test (2) Seliwanoff’s test
(3) Barfoed’s test (4) Benedict’s test
144. The major product of the following reaction is

(1) (2)

Corporate Office : Aakash Tower, 8, Pusa Road, New Delhi-110005 [31]

For More Material Join: @JEEAdvanced_2024


For More Material Join: @JEEAdvanced_2024
Unique Practice Questions Chemistry

(3) (4)

145. The peptide that gives positive ceric ammonium nitrate and carbylamine tests is
(1) Ser - Lys (2) Lys - Asp
(3) Gln - Asp (4) Asp – Gln
146. Consider the given plot of enthalpy of the following reaction between A and B.
A +B → C+D
Identify the incorrect statement.

(1) Activation enthalpy to form C is 5 kJ mol–1 less than that to form D


(2) D is kinetically stable product
(3) Formation of A and B from C has highest enthalpy of activation
(4) C is the thermodynamically stable product
147. The amorphous form of silica is
(1) Quartz (2) Tridymite
(3) Kieselguhr (4) Cristobalite
148. The principle of column chromatography is :
(1) Differential adsorption of the substances on the solid phase.
(2) Gravitational force.
(3) Differential absorption of the substances on the solid phase.
(4) Capillary action.
149. For a reaction at equilibrium

1
A(g) B(g) + C(g)
2

The relation between dissociation constant (K), degree of dissociation (α) and equilibrium pressure (p) is given
by:
1 3
 2p2
K= 3 1
1  2p2
 3  2 K=
 1 + 2   (1 −  )
1

(1)   (2) (2 +  ) 2 (1 −  )

Corporate Office : Aakash Tower, 8, Pusa Road, New Delhi-110005 [32]

For More Material Join: @JEEAdvanced_2024


For More Material Join: @JEEAdvanced_2024
Unique Practice Questions Chemistry

3
(p) 2
K= 3
1 (p) 2
 3  2 K=
 1 + 2   (1 −  )
1

(3)   (4) (1 +  )(1 −  ) 2

150. Given below are two statements:

Statement I : Emulsion of oil in water are unstable and sometimes they separate into two layers on standing.
Statement II : For stabilisation of an emulsion, excess of electrolyte is added.
In the light of the above statements, choose the most appropriate answer from the options given below:
(1) Both Statement I and Statement II are correct
(2) Both Statement I and Statement II are incorrect.
(3) Statement I is correct but Statement II is incorrect.
(4) Statement I is incorrect but Statement II is correct.
151. In the given reaction sequence, the major product ‘C’ is:

(1) (2)

(3) (4)

152. Which of the following is an example of polyester?


(1) Butadiene-styrene copolymer
(2) Melamine polymer
(3) Neoprene
(4) Poly-β-hydroxybutyrate-co-β-hydroxy valerate
153. A polysaccharide ‘X’ on boiling with dil. H2SO4 at 393 K under 2-3 atm pressure yields ‘Y’. ‘Y’ on treatment with
bromine water gives gluconic acid. ‘X’ contains β-glycosidic linkages only. Compound ‘X’ is:
(1) Starch (2) Cellulose
(3) Amylose (4) Amylopectin
154. Which of the following is not a broad-spectrum antibiotic?
(1) Vancomycin (2) Ampicillin
(3) Ofloxacin (4) Penicillin G
155. Which one of the following elemental forms is not present in the enamel of the teeth?

Corporate Office : Aakash Tower, 8, Pusa Road, New Delhi-110005 [33]

For More Material Join: @JEEAdvanced_2024


For More Material Join: @JEEAdvanced_2024
Unique Practice Questions Chemistry

(1) Ca2+ (2) P3+


(3) F– (4) P5+
156. Match List-I with List -II :
List-I List-II
(A) Sphalerite (I) FeCO3
(B) Calamine (II) PbS
(C) Galena (III) ZnCO3
(D) Siderite (IV) ZnS
Choose the most appropriate answer from the options given below:
(1) (A)-(IV), (B)-(III), (C)-(II), (D)-(I) (2) (A)-(IV), (B)-(I), (C)-(II), (D)-(III)
(3) (A)-(II), (B)-(III), (C)-(I), (D)-(IV) (4) (A)-(III), (B)-(IV), (C)-(II), (D)-(I)
157. Metals generally melt at very high temperature. Amongst the following, the metal with the highest melting point
will be
(1) Hg (2) Ag
(3) Ga (4) Cs
158. Which of the following chemical reactions represents Hall-Heroult Process?
(1) Cr2O3 + 2Al → Al2O3 + 2Cr (2) 2Al2O3 + 3C → 4Al + 3CO2

2Au(CN)2 (aq) + Zn(s) → 2Au(s) + [Zn(CN4 )]2–


(3) FeO + CO → Fe + CO2 (4)

159. In the industrial production of which of the following, molecular hydrogen is obtained as a byproduct?
(1) NaOH (2) NaCl
(3) Na metal (4) Na2CO3
160. Some gases are responsible for heating of atmosphere (green house effect). Identify from the following the
gaseous species which does not cause it.
(1) CH4 (2) O3
(3) H2O (4) N2
161. Which of the following is not an example of a condensation polymer?
(1) Nylon 6,6 (2) Dacron
(3) Buna-N (4) Silicone
162. The structure shown below is of which well-known drug molecule?

(1) Ranitidine (2) Seldane

Corporate Office : Aakash Tower, 8, Pusa Road, New Delhi-110005 [34]

For More Material Join: @JEEAdvanced_2024


For More Material Join: @JEEAdvanced_2024
Unique Practice Questions Chemistry

(3) Cimetidine (4) Codeine


163. In the flame test of a mixture of salts, a green flame with blue centre was observed. Which one of the following
cations may be present?
(1) Cu2+ (2) Sr2+
(3) Ba2+ (4) Ca2+
164. (a) Baryte, (b) Galena, (c) Zinc blende and (d) Copper pyrites. How many of these minerals are sulphide
based?
165. In alanylglycylleucylalanyvaline, the number of peptide linkages is __________.,
166. Incorrect statement for Tyndall effect is :
(1) The refractive indices of the dispersed phase and the dispersion medium differ greatly in magnitude.
(2) The diameter of the dispersed particles is much smaller than the wavelength of the light used.
(3) During projection of movies in the cinemas hall, Tyndall effect is noticed.
(4) It is used to distinguish a true solution from a colloidal solution.
167. The eutrophication of water body results in:
(1) loss of Biodiversity. (2) breakdown of organic matter.
(3) increase in biodiversity. (4) decrease in BOD.
168. Using very little soap while washing clothes, does not serve the purpose of cleaning of clothes, because:
(1) soap particles remain floating in water as ions.
(2) the hydrophobic part of soap is not able to take away grease.
(3) the micelles are not formed due to concentration of soap, below its CMC value.
(4) colloidal structure of soap in water is completely distributed.
169. Which one of the following is an example of artificial sweetner?
(1) Bithional (2) Alitame
(3) Salvarsan (4) Lactose
170. For a given chemical reaction
γ1A + γ2B → γ3C + γ4D
Concentration of C changes from 10 mmol dm –3 to 20 mmol dm–3 in 10 seconds. Rate of appearance of D is
1.5 times the rate of disappearance of B which is twice the rate of disappearance A. The rate of appearance
of D has been experimentally determined to be 9 mmol dm–3 s–1. Therefore, the rate of reaction is _____ mmol
dm–3 s–1.
(Nearest Integer)
171. Match List I with List II.
List I List II
A Zymase I Stomach
B Diastase II Yeast
C Urease III Malt

Corporate Office : Aakash Tower, 8, Pusa Road, New Delhi-110005 [35]

For More Material Join: @JEEAdvanced_2024


For More Material Join: @JEEAdvanced_2024
Unique Practice Questions Chemistry

D Pepsin IV Soyabean
Choose the correct answer from the options given below
(1) A-II, B-III, C-I, D-IV
(2) A-II, B-III, C-IV, D-I
(3) A-III, B-II, C-IV, D-I
(4) A-III, B-II, C-I, D-IV
172. Given below are two statements.
Statement-I: During electrolytic refining, blister copper deposits precious metals.
Statement-II: In the process of obtaining pure copper by electrolysis method, copper blister is used to make
the anode.
In the light of the above statements, choose the correct answer from the options given below.
(1) Both Statement-I and Statement-II are true.
(2) Both Statement-I and Statement-II are false.
(3) Statement-I is true but Statement II is false.
(4) Statement-I is false but Statement-II is true
173. The number of bridged oxygen atoms present in compound B formed from the following reactions is
Pb(NO3 )2 ⎯⎯⎯
673 K
→ A + PbO + O2

A ⎯⎯⎯⎯
Dimerise
→B
(1) 0 (2) 1
(3) 2 (4) 3
174. Given below are two statements: one is labelled as Assertion A and the other is labelled as Reason R.
Assertion A: Polluted water may have a value of BOD of the order of 17 ppm.
Reason R: BOD is a measure of oxygen required to oxidise both the bio-degradable and non-biodegradable
organic material in water.
In the light of the above statements, choose the most appropriate answer from the options given below.
(1) Both A and R are correct and R is the correct explanation of A.
(2) Both A and R are correct but R is NOT the correct explanation of A.
(3) A is correct but R is not correct.
(4) A is not correct but R is correct.
175. Given below are two statements: one is labelled as Assertion A and the other is labelled as Reason R.
Assertion A: A mixture contains benzoic acid and naphthalene. The pure benzoic acid can be separated out
by the use of benzene.
Reason R: Benzoic acid is soluble in hot water.
In the light of the above statements, choose the most appropriate answer from the options given below.
(1) Both A and R are true and R is the correct explanation of A.
(2) Both A and R are true but R is NOT the correct explanation of A.
(3) A is true but R is false.
(4) A is false but R is true.

Corporate Office : Aakash Tower, 8, Pusa Road, New Delhi-110005 [36]

For More Material Join: @JEEAdvanced_2024


For More Material Join: @JEEAdvanced_2024
Unique Practice Questions Chemistry

176. An antiseptic Dettol is a mixture of two compounds ‘A’ and ‘B’ where A has 6 electrons and B has 2 electrons.
What is ‘B’?
(1) Bithionol (2) Terpineol
(3) Chloroxylenol (4) Chloramphenicol
177. How many of the given compounds will give a positive Biuret test_________? Glycine, Glycylalanine,
Tripeptide, Biuret.
 E 
178. The   of different types of half cells are as follows:
 T P
A B C D
−4 −4 −4
1 10 2  10 0.1 10 0.2  10−4
(Where E is the electromotive force)
Which of the above half cells would be preferred to be used as reference electrode?
(1) A (2) B
(3) C (4) D
179. Given below are two statements:
Statement I : According to the Ellingham diagram, any metal oxide with higher G° is more stable than the
one with lower G°.
Statement II : The metal involved in the formation of oxide placed lower in the Ellingham diagram can reduce
the oxide of a metal placed higher in the diagram.
In the light of the above statements, choose the most appropriate answer from the options given below:
(1) Both Statement I and Statement II are correct.
(2) Both Statement I and Statement II are incorrect.
(3) Statement I is correct but Statement II is incorrect.
(4) Statement I is incorrect but Statement II is correct.
180. Polar stratospheric clouds facilitate the formation of:
(1) ClONO2 (2) HOCl
(3) ClO (4) CH4
181. Given below are two statements:
Statement I: In ‘Lassaigne’s Test’, when both nitrogen and sulphur are present in an organic compound,
sodium thiocyanate is formed.
Statement II: If both nitrogen and sulphur are present in an organic compound, then the excess of sodium
used in sodium fusion will decompose the sodium thiocyanate formed to give NaCN and Na 2S.
In the light of the above statements, choose the most appropriate answer from the options given below:
(1) Both Statement I and Statement II are correct.
(2) Both Statement I and Statement II are incorrect.
(3) Statement I is correct but Statement II is incorrect.
(4) Statement I is incorrect but Statement II is correct.
182. Which of the following sets are correct regarding polymer?

Corporate Office : Aakash Tower, 8, Pusa Road, New Delhi-110005 [37]

For More Material Join: @JEEAdvanced_2024


For More Material Join: @JEEAdvanced_2024
Unique Practice Questions Chemistry

(A) Copolymer : Buna-S


(B) Condensation polymer : Nylon-6,6
(C) Fibres : Nylon-6,6
(D) Thermosetting polymer : Terylene
(E) Homopolymer : Buna-N
Choose the correct answer from given options below:
(1) (A), (B) and (C) are correct (2) (B), (C) and (D) are correct
(3) (A), (C) and (E) are correct (4) (A), (B) and (D) are correct
183. A chemical which stimulates the secretion of pepsin is:
(1) Anti-histamine (2) Cimetidine
(3) Histamine (4) Zantac
184. Which statement is not true with respect to nitrate ion test?
(1) A dark brown ring is formed at the junction of two solutions.
(2) Ring is formed due to nitroferrous sulphate complex.
(3) The brown complex is [Fe(H2O)5 (NO)]SO4.
(4) Heating the nitrate salt with conc. H2SO4, light brown fumes are evolved.
185. The number of oxygens present in a nucleotide formed from a base, that is present only in RNA is ________.
186. Match List I with List II
List I List II
Enzyme Conversion of
A Invertase I Starch into maltose
B Zymase II Maltose into glucose
C Diastase III Glucose into ethanol
D Maltase IV Cane sugar into
glucose
Choose the most appropriate answer from the options given below
(1) A-III, B-IV, C-II, D-I (2) A-III, B-II, C-I, D-IV
(3) A-IV, B-III, C-I, D-II (4) A-IV, B-II, C-III, D-I
187. s-block element which cannot be qualitatively confirmed by the flame test is
(1) Li (2) Na
(3) Rb (4) Be
188. The measured BOD values for four different water samples (A-D) are as follows:
A = 3 ppm; B = 18 ppm; C = 21 ppm; D = 4 ppm;. The water samples which can be called as highly polluted
with organic wastes, are
(1) A and B (2) A and D
(3) B and C (4) B and D
189. Which one of the following is a water soluble vitamin, that is not excreted easily?

Corporate Office : Aakash Tower, 8, Pusa Road, New Delhi-110005 [38]

For More Material Join: @JEEAdvanced_2024


For More Material Join: @JEEAdvanced_2024
Unique Practice Questions Chemistry

(1) Vitamin B2 (2) Vitamin B1


(3) Vitamin B6 (4) Vitamin B12
190. Match List-I with List-II
List-I List-II
(A) Lyophilic colloid (I) Liquid-liquid colloid
(B) Emulsion (II) Protective colloid
(C) Positively charged (III) FeCl3 + NaOH colloid
(D) Negatively charged (IV) FeCl3 + hot water colloid
Choose the correct answer from the options given below :
(1) (A)-(II), (B)-(I), (C)-(IV), (D)-(III) (2) (A)-(III), (B)-(I), (C)-(IV), (D)-(II)
(3) (A)-(II), (B)-(I), (C)-(III), (D)-(IV) (4) (A)-(III), (B)-(II), (C)-(I), (D)-(IV)
191. Match List-I with List-II.
List-I List-II
(A) Concentration of (I) Aniline Gold ore
(B) Leaching of alumina (II) NaOH
(C) Froth stabiliser (III) SO2
(D) Blister copper (IV) NaCN
Choose the correct answer from the options given below.
(1) (A)-(IV), (B)-(III), (C)-(II), (D)-(I) (2) (A)-(IV), (B)-(II), (C)-(I), (D)-(III)
(3) (A)-(III), (B)-(II), (C)-(I), (D)-(IV) (4) (A)-(II), (B)-(IV), (C)-(III), (D)-(I)
192. Given below are two Statements:
Statement I: Classical smog occurs in cool humid climate. It is a reducing mixture of smoke, fog and sulphur
dioxide.
Statement II: Photochemical smog has components, ozone, nitric oxide, acrolein, formaldehyde, PAN etc.
In the light of the above statements, choose the most appropriate answer from the options given below.
(1) Both Statement I and Statement II are correct.
(2) Both Statement I and Statement II are incorrect.
(3) Statement I is correct but Statement II is incorrect.
(4) Statement I is incorrect but Statement II is correct
193. Which of the following is structure of a separating funnel?

(1) (2)

Corporate Office : Aakash Tower, 8, Pusa Road, New Delhi-110005 [39]

For More Material Join: @JEEAdvanced_2024


For More Material Join: @JEEAdvanced_2024
Unique Practice Questions Chemistry

(3) (4)

194. Match List-I with List-II


List-I List-II
(Polymer) (Used in)
(A) Bakelite (I) Radio and television cabinets
(B) Glyptal (II) Electrical switches
(C) PVC (III) Paints and Lacqures
(D) Polystyrene (IV) Water pipes
Choose the correct answer from the options given below:
(1) (A) –(II) (B) – (III), (C) – (IV), (D) – (I) (2) (A) –(I) (B) – (II), (C) – (III), (D) – (IV)
(3) (A) –(IV) (B) – (III), (C) – (II), (D) – (I) (4) (A) –(II) (B) – (III), (C) – (I), (D) – (IV)
195. Match List-I with List-II
List-I List-II

(A) (I) Dishwashing power

(B) (II) Toothpaste

(C) C17H35COO–Na++Na2CO3+Rosinate (III) Laundry soap


(D) CH3(CH2)16COO(CH2CH2O)CH2CH2OH (IV) Hair conditional
Choose the correct answer from the options given below:
(1) (A) –(III) (B) – (II), (C) – (IV), (D) – (I) (2) (A) –(IV) (B) – (II), (C) – (III), (D) – (I)
(3) (A) –(IV) (B) – (III), (C) – (II), (D) – (I) (4) (A) –(III) (B) – (IV), (C) – (I), (D) – (II)
196. Statement-I : Leaching of gold with cyanide ion in absence of air/O 2 leads to cyano complex of Au(III).
Statement-II : Zinc is oxidized during the displacement reaction carried out for gold extraction.
In the light of the above statements, choose the correct answer from the options given below.
(1) Both statement-I and statement-II are correct
(2) Both statement-I and statement-II are incorrect
(3) Statement-I is correct but statement-II is incorrect
(4) Statement-I is incorrect but statement-II is correct
197. The f orbitals are half and completely filled, respectively in lanthanide ions
[Given: Atomic no. Eu, 63; Sm, 62; Tm, 69; Tb, 65; Yb, 70; Dy, 66]
(1) Eu2+ and Tm2+ (2) Sm2+ and Tm3+

Corporate Office : Aakash Tower, 8, Pusa Road, New Delhi-110005 [40]

For More Material Join: @JEEAdvanced_2024


For More Material Join: @JEEAdvanced_2024
Unique Practice Questions Chemistry

(3) Tb4+ and Yb2+ (4) Dy3+ and Yb3+


198. Which is true about Buna-N?
(1) It is a linear polymer of 1, 3-butadiene
(2) It is obtained by copolymerization of 1, 3-butadiene and styrene
(3) It is obtained by copolymerization of 1, 3-butadiene and acrylonitrile
(4) The suffix N in Buna-N stands for its natural occurrence.
199. Given below are two statements
Statement I: Maltose has two -D-glucose units linked at C1 and C4 and is a reducing sugar.
Statement II: Maltose has two monosaccharides: -D-glucose and -D-glucose linked at C1 and C6 and it is
a non-reducing sugar.
In the light of the above statements, choose the correct answer from the options given below.
(1) Both Statement I and Statement II are true
(2) Both Statement I and Statement II are false
(3) Statement I is true but Statement II is false
(4) Statement I is false but Statement II is true
200. Match List I with List II.
List I List II
A. Antipyretic I. Reduces pain
B. Analgesic II. Reduces stress
C. Tranquilizer III. Reduces fever
D. Antacid IV. Reduces acidity
(stomach)
Choose the correct answer from the options given below:
(1) A-III, B-I, C-II, D-IV
(2) A-III, B-I, C-IV, D-II
(3) A-I, B-IV, C-II, D-III
(4) A-I, B-III, C-II, D-IV
201. Given are two statements one is labelled as Assertion A and other is labelled as Reason R.
Assertion A : Magnesium can reduce Al2O3 at a temperature below 1350°C, while above 1350°C aluminium
can reduce MgO.
Reason R : The melting and boiling points of magnesium are lower than those of aluminium.
In light of the above statements, choose most appropriate answer from the options given below :
(1) Both A and R are correct, and R is correct explanation of A.
(2) Both A and R are correct, but R is NOT the correct explanation of A.
(3) A is correct R is not correct.
(4) A is not correct, R is correct.
202. Which one of the lanthanoids given below is the most stable in divalent form?

Corporate Office : Aakash Tower, 8, Pusa Road, New Delhi-110005 [41]

For More Material Join: @JEEAdvanced_2024


For More Material Join: @JEEAdvanced_2024
Unique Practice Questions Chemistry

(1) Ce (Atomic Number 58) (2) Sm (Atomic Number 62)


(3) Eu (Atomic Number 63) (4) Yb (Atomic Number 70)
203. The formula of the purple colour formed in Laissaigne’s test for sulphur using sodium nitroprusside is
(1) NaFe[Fe(CN)6] (2) Na[Cr(NH3)2(NCS)4]
(3) Na2[Fe(CN)5(NO)] (4) Na4[Fe(CN)5(NOS)]
204. Which one of the following techniques is not used to spot components of a mixture separated on thin layer
chromatographic plate?
(1) I2 (Solid)
(2) U.V. Light
(3) Visualisation agent as a component of mobile phase
(4) Spraying of an appropriate reagent
205. Which amongst the following is not a pesticide?
(1) DDT (2) Organophosphates
(3) Dieldrin (4) Sodium arsenite
206. Match List-I with List-II :
List-I List-II
(A) Negatively charged sol (I) Fe2O3  xH2O

(B) Macromolecular colloid (II) CdS sol


(C) Positively charged sol (III) Starch
(D) Cheese (IV) a gel
Choose the correct answer from the options given below:
(1) (A) – (II), (B) – (III), (C) – (IV), (D) – (I) (2) (A) – (II), (B) – (I), (C) – (III), (D) – (IV)
(3) (A) – (II), (B) – (III), (C) – (I), (D) – (IV) (4) (A) – (I), (B) – (III), (C) – (II), (D) – (IV)
207. In the metallurgical extraction of copper, following reaction is used :
FeO + SiO2 → FeSiO3
FeO and FeSiO3 respectively are.
(1) Gangue and flux (2) Flux and slag
(3) Slag and flux (4) Gangue and slag
208. Given below are two statements, one is Assertion (A) and other is Reason (R).
Assertion (A): Natural rubber is a linear polymer of isoprene called cis-polyisoprene with elastic properties.
Reason (R): The cis-polyisoprene molecules consist of various chains held together by strong polar
interactions with coiled structure.
In the light of the above statements, choose the correct one from the options given below:
(1) Both (A) and (R) are true and (R) is the correct explanation of (A).
(2) Both (A) and (R) are true but (R) is not the correct explanation of (A).
(3) (A) is true but (R) is false.
(4) (A) is false but (R) is true.

Corporate Office : Aakash Tower, 8, Pusa Road, New Delhi-110005 [42]

For More Material Join: @JEEAdvanced_2024


For More Material Join: @JEEAdvanced_2024
Unique Practice Questions Chemistry

209. When sugar ‘X’ is boiled with dilute H2SO4 in alcoholic solution, two isomers ‘A’ and ‘B’ are formed. ‘A’ on
oxidation with HNO3 yields saccharic acid whereas ‘B’ is laevorotatory. The compound ‘X’ is :
(1) Maltose (2) Sucrose
(3) Lactose (4) Starch
210. The drug tegamet is:

(1)

(2)

(3)

(4)

211. Which of the following is a correct statement?


(1) Brownian motion destabilises sols.
(2) Any amount of dispersed phase can be added to emulsion without destabilising it.
(3) Mixing two oppositely charged sols in equal amount neutralises charges and stabilises colloids.
(4) Presence of equal and similar charges on colloidal particles provides stability to the colloidal solution.
212. Which one of following reactions indicates the reducing ability of hydrogen peroxide in basic medium?
(1) HOCI + H2O2 → H3O+ + CI– + O2
(2) PbS + 4H2O2 → PbSO4 + 4H2O
(3) 2MnO4– + 3H2O2 → 2MnO2 + 3O2 + 2H2O + 2OH–

(4) Mn2+ + H2O2 → Mn4+ + 2OH–


213. The acid that is believed to be mainly responsible for the damage of Taj Mahal is
(1) Sulfuric acid (2) Hydrofluoric acid
(3) Phosphoric acid (4) Hydrochloric acid
214. The polymer, which can be stretched and retains its original status on releasing the force is
(1) Bakelite (2) Nylon 6, 6
(3) Buna-N (4) Terylene
215. Sugar moiety in DNA and RNA molecules respectively are

Corporate Office : Aakash Tower, 8, Pusa Road, New Delhi-110005 [43]

For More Material Join: @JEEAdvanced_2024


For More Material Join: @JEEAdvanced_2024
Unique Practice Questions Chemistry

(1) -D-2-deoxyribose, -D-deoxyribose (2) -D-2-deoxyribose, -D-ribose


(3) -D-ribose, -D-2-deoxyribose (4) -D-deoxyribose, -D-2-deoxyribose
216. Which of the following compound does not contain sulphur atom?
(1) Cimetidine (2) Ranitidine
(3) Histamine (4) Saccharin
217. The number of terminal oxygen atoms present in the product B obtained from the following reaction is
________.
FeCr2O4 + Na2CO3 + O2 → A + Fe2O3 + CO2
A + H+ → B + H2O + Na+
218. Match List-I with List-II
List-I Ore List-II Composition
A. Siderite I. FeCO3
B. Malachite II. CuCO3. Cu(OH)2
C. Sphalerite III. ZnS
D. Calamine IV. ZnCO3
Choose the correct answer from the options given below:
(1) A-I, B-II, C-III, D-IV (2) A-III, B-IV, C-II, D-I
(3) A-IV, B-III, C-I, D-II (4) A-I, B-II, C-IV, D-III
219. Given below are two statements : one is labelled as Assertion A and the other is labelled as Reason R.
Assertion A : Dacron is an example of polyester polymer.
Reason R : Dacron is made up of ethylene glycol and terephthalic acid monomers.
In the light of the above statements, choose the most appropriate answer from the options given below.
(1) Both A and R are correct and R is the correct explanation of A.
(2) Both A and R are correct but R is NOT the correct explanation of A.
(3) A is correct but R is not correct.
(4) A is not correct but R is correct.
220. The structure of protein that is unaffected by heating is
(1) Secondary Structure (2) Tertiary Structure
(3) Primary Structure (4) Quaternary Structure
221. The mixture of chloroxylenol and terpineol is an example of
(1) Antiseptic (2) Pesticide
(3) Disinfectant (4) Narcotic analgesic
222. The photochemical smog does not generally contain :
(1) NO (2) NO2
(3) SO2 (4) HCHO
223. During the denaturation of proteins, which of these structures will remain intact?
(1) Primary (2) Secondary
(3) Tertiary (4) Quaternary

Corporate Office : Aakash Tower, 8, Pusa Road, New Delhi-110005 [44]

For More Material Join: @JEEAdvanced_2024


For More Material Join: @JEEAdvanced_2024
Unique Practice Questions Chemistry

224. Drugs used to bind to receptors, inhibiting its natural function and blocking a message are called:
(1) Agonists (2) Antagonists
(3) Allosterists (4) Anti histaminists
225. Given below are two statements:
Statement I: On heating with KHSO4, glycerol is dehydrated and acrolein is formed.
Statement II: Acrolein has fruity odour and can be used to test glycerol’s presence.
Choose the correct option.
(1) Both Statement I and Statement II are correct.
(2) Both Statement I and Statement II are incorrect.
(3) Statement I is correct but Statement II is incorrect.
(4) Statement I is incorrect but Statement II is correct.
226. For micelle formation, which of the following statements are correct?
A. Micelle formation is an exothermic process.
B. Micelle formation is an endothermic process.
C. The entropy change is positive
D. The entropy change is negative
(1) A and D only (2) A and C only
(3) B and C only (4) B and D only
227. Given below are two statements.
Statement-I : Pig iron is obtained by heating cast iron with scrap iron.
Statement-II : Pig iron has a relatively lower carbon content than that of cast iron.
In the light of the above statements, choose the correct answer from the options given below.
(1) Both Statement-I and Statement-II are correct
(2) Both Statement-I and Statement-II are not correct.
(3) Statement-I is correct but Statement-II is not correct
(4) Statement-I is not correct but Statement-II is correct
228. High purity (>99.95%) dihydrogen is obtained by
(1) Reaction of zinc with aqueous alkali
(2) Electrolysis of acidified water using platinum electrodes
(3) Electrolysis of warm aqueous barium hydroxide solution between nickel electrodes
(4) Reaction of zinc with dilute acid
229. Match List I with List II.
List I List II
A. Sulphate I. Pesticide
B. Fluoride II. Bending of bones
C. Nicotine III. Laxative effect
D. Sodium arsinite IV. Herbicide
Choose the correct answer from the options given below:
(1) A-II, B-III, C-IV, D-I (2) A-IV, B-III, C-II, D-I

Corporate Office : Aakash Tower, 8, Pusa Road, New Delhi-110005 [45]

For More Material Join: @JEEAdvanced_2024


For More Material Join: @JEEAdvanced_2024
Unique Practice Questions Chemistry

(3) A-III, B-II, C-I, D-IV (4) A-III, B-II, C-IV, D-I
230. Match List I with List II:
List I List II
Polymer Used for items
A. Nylon 6, 6 I Buckets
B. Low density II. Non-stick utensils polythene
C. High density III. Bristles of brushes polythene
D. Teflon IV. Toys
Choose the correct answer from the options given below:
(1) A-III, B-I, C-IV, D-II (2) A-III, B-IV, C-I, D-II
(3) A-II, B-I, C-IV, D-III (4) A-II, B-IV, C-I, D-III
231. Glycosidic linkage between C1 of -glucose and C2 of -fructose is found in
(1) Maltose (2) Sucrose
(3) Lactose (4) Amylose
232. Some drugs bind to a site other than the active site of an enzyme. This site is known as
(1) Non-active site (2) Allosteric site
(3) Competitive site (4) Therapeutic site
233. Which of the following can be used to prevent the decomposition of H2O2?
(1) Urea (2) Formaldehyde
(3) Formic acid (4) Ethanol
234. Which technique among the following, is most appropriate in separation of a mixture of 100 mg of p-nitrophenol
and picric acid?
(1) Steam distillation
(2) 2-5 ft long column of silica gel
(3) Sublimation
(4) Preparative TLC (Thin Layer Chromatography)
235. Stearic acid and polyethylene glycol react to form which one of the following soap/s detergents?
(1) Cationic detergent (2) Soap
(3) Anionic detergent (4) Non-ionic detergent
236. Given below are two statements: one is labelled as Assertion A and other is labelled as Reason R.
Assertion A : Finest gold is red in colour, as the size of the particles increases, it appears purple then blue
and finally gold.
Reason R : The colour of the colloidal solution depends on the wavelength of light scattered by the dispersed
particles.
In the light of the above statements, choose the most appropriate answer from the options given below.
(1) Both A and R are true and R is the correct explanation of A
(2) Both A and R are true but R is NOT the correct explanation of A

Corporate Office : Aakash Tower, 8, Pusa Road, New Delhi-110005 [46]

For More Material Join: @JEEAdvanced_2024


For More Material Join: @JEEAdvanced_2024
Unique Practice Questions Chemistry

(3) A is true but R is false


(4) A is false but R is true
237. Match List I with List II.
List I List II
Pollutant Source
A. Microorganisms I. Strip mining
B. Plant nutrients II. Domestic sewage
C. Toxic heavy metals III. Chemical fertilizer
D. Sediment IV. Chemical factory
Choose the correct answer from the options given below:
(1) A-II, B-III, C-IV, D-I (2) A-II, B-I, C-IV, D-III
(3) A-I, B-IV, C-II, D-III (4) A-I, B-IV, C-III, D-II
238. Vulcanization of rubber is carried out by heating a mixture of
(1) Isoprene and styrene
(2) Neoprene and sulphur
(3) Isoprene and sulphur
(4) Neoprene and styrene
239. Animal starch is the other name of
(1) Amylose (2) Maltose
(3) Glycogen (4) Amylopectin
240. How many of the following drugs is/are examples(s) of broad-spectrum antibiotics?
Ofloxacin, Penicillin G, Terpineol, Salvarsan.
241. Given below are two statements. One is labelled as Assertion A and the other is labelled as Reason R.
Assertion A: Activated charcoal adsorbs SO2 more efficiently than CH4.
Reason R: Gases with lower critical temperatures are readily adsorbed by activated charcoal.
In the light of the above statements, choose the correct answer from the options given below.
(1) Both A and R are correct and R is the correct explanation of A.
(2) Both A and R are correct but R is NOT the correct explanation of A.
(3) A is correct but R is not correct.
(4) A is not correct but R is correct.
242. Which of the following methods are not used to refine any metal?
A. Liquation
B. Calcination
C. Electrolysis
D. Leaching
E. Distillation
Choose the correct answer from the options given below :
(1) B and D only (2) A, B, D and E only

Corporate Office : Aakash Tower, 8, Pusa Road, New Delhi-110005 [47]

For More Material Join: @JEEAdvanced_2024


For More Material Join: @JEEAdvanced_2024
Unique Practice Questions Chemistry

(3) B, D and E only (4) A, C and E only


243. Match List I with List II.
List I List II
Pollutant Disease/ sickness
A. Sulphate I. Methemoglobinemia
(> 500 ppm)
B. Nitrate II. Brown mottling of teeth
(> 50 ppm)
C. Lead (> 50 ppb) III. Laxative effect
D. Fluoride IV. Kidney damage
(> 2ppm)
Choose, the coned answer from the options given below:
(1) A-IV, B-I, C-II, D-III (2) A-III, B-I, C-IV, D-II
(3) A-II, B-IV, C-I, D-III (4) A-II, B-IV, C-III, D-I
244. Match List-I with List-II.
List-1 List II
Polymers Commercial
names
A. Phenol- I. Glyptal
formaldehyde resin
B. Copolymer of II. Novolac
1,3-butadiene and
styrene
C. Polyester of glycol III. Buna-S
and phthalic acid
D. Polyester of glycol IV. Dacron
and terephthalic acid
Choose the correct answer from the option give below:
(1) A-II, B-III, C-IV, D-I (2) A-II, B-III, C-I, D-IV
(3) A-II, B-I, C-III, D-IV (4) A-III, B-II, C-IV, D-I
245. Match List I and List II.

Corporate Office : Aakash Tower, 8, Pusa Road, New Delhi-110005 [48]

For More Material Join: @JEEAdvanced_2024


For More Material Join: @JEEAdvanced_2024
Unique Practice Questions Chemistry

Choose the correct answer from the options given below:


(1) A-IV, B-III, C-II, D-I (2) A-III, B-I, C-II, D-IV
(3) A-III, B-IV, C-I, D-II (4) A-III, B-I, C-IV, D-II
246. Given below are two statements : one is labelled as Assertion (A) and the other is labelled as Reason (R) :
Assertion (A) : Dissolved substances can be removed from a colloidal solution by diffusion through a
parchment paper.
Reason (R) : Particles in a true solution cannot pass through parchment paper but the colloidal particles can
pass through the parchment paper.
In the light of the above statements, choose the correct answer from the options given below
(1) Both (A) and (R) are correct and (R) is the correct explanation of (A)
(2) Both (A) and (R) are correct but (R) is not the correct explanation of (A)
(3) (A) is correct but (R) is not correct
(4) (A) is not correct but (R) is correct
247. Given below are two statements:
Statement I : The non bio-degradable fly ash and slag from steel industry used by cement industry.
Statement II : The fuel obtained from plastic waste is lead free.
In the light of the above statements, choose the most appropriate answer from the options given below:
(1) Both Statement I and Statement II are correct

Corporate Office : Aakash Tower, 8, Pusa Road, New Delhi-110005 [49]

For More Material Join: @JEEAdvanced_2024


For More Material Join: @JEEAdvanced_2024
Unique Practice Questions Chemistry

(2) Both Statement I and Statement II are incorrect


(3) Statement I is correct but Statement II is incorrect
(4) Statement I is incorrect but Statement II is correct
248. Match List-I with List-II
List-I (Polymer) List-II (Monomer)
(1) Neoprene (I) Acrylonitrile
(B) Teflon (II) Chloroprene
(C) Acrilan (III) Tetrafluroethene
(D) Natural rubber (IV) Isoprene
Choose the correct answer from the options given below:
(1) (A) - (II), (B) – (III), (C) – (I), (D) – (IV) (2) (A) - (II), (B) – (I), (C) – (III), (D) – (IV)
(3) (A) - (II), (B) – (I), (C) – (IV), (D) – (III) (4) (A) - (I), (B) – (II), (C) – (III), (D) – (IV)
249. Match List-I with List-II
List-I (Mixture) List-II (Purification Process)
(A) Chloroform & Aniline (I) Steam distillation
(B) Benzoic acid & (II) Sublimation Napthalene
(C) Water & Aniline (III) Distillation
(D) Napthalene & Sodium (IV) Crystallisation chloride
Choose the correct answer from the options given below:
(1) (A)-(IV), (B)-(III), (C)-(I), (D)-(II)
(2) (A)-(III), (B)-(I), (C)-(IV), (D)-(II)
(3) (A)-(III), (B)-(IV), (C)-(II), (D)-(I)
(4) (A)-(III), (B)-(IV), (C)-(I), (D)-(II)
250. Among the following ores Bauxite, Siderite, Cuprite, Calamine, Haematite, Kaolinite, Malachite, Magnetite,
Sphalerite, Limonite, Cryolite, the number of principal ores of iron is____.
251. Given below are two statements:
Statement I: In polluted water values of both dissolved oxygen and BOD are very low.
Statement II: Eutrophication results in decrease in the amount of dissolved oxygen.
In the light of the above statements, choose the most appropriate answer from the options given below:
(1) Both Statement I and Statement II are true
(2) Both Statement I and Statement II are false
(3) Statement I is true but Statement II is false
(4) Statement I is false but Statement II is true
252. Terylene polymer is obtained by condensation of:
(1) Ethane-1, 2-diol and Benzene-1, 3 dicarboxylic acid
(2) Propane-1, 2-diol and Benzene-1, 4 dicarboxylic acid
(3) Ethane-1, 2-diol and Benzene-1, 4 dicarboxylic acid
(4) Ethane-1, 2-diol and Benzene-1, 2 dicarboxylic acid

Corporate Office : Aakash Tower, 8, Pusa Road, New Delhi-110005 [50]

For More Material Join: @JEEAdvanced_2024


For More Material Join: @JEEAdvanced_2024
Unique Practice Questions Chemistry

253. Statements about Enzyme Inhibitor Drugs are given below :


(1) There are Competitive and Non-competitive inhibitor drugs.
(2) These can bind at the active sites and allosteric sites.
(3) Competitive Drugs are allosteric site blocking drugs.
(4) Non-competitive Drugs are active site blocking drugs.
Choose the correct answer from the options given below:
(1) (A), (D) only (2) (A), (C) only
(3) (A), (B) only (4) (A), (B), (C) only
254. Statement-I: An alloy of lithium and magnesium is used to make aircraft plates.
Statement-II: The magnesium ions are important for cell-membrane integrity.
In the light of the above statements, choose the correct answer from the options given below:
(1) Both Statement-I and Statement-II are true
(2) Both Statement-I and Statement-II are false
(3) Statement-I is true but Statement-II is false
(4) Statement-I is false but Statement-II is true
255. Match List-I with List-II

Choose the correct answer from the options given below:


(1) A-II, B-III, C-IV, D-I (2) A-III, B-II, C-IV, D-I
(3) A-III, B-I, C-IV, D-II (D) A-I, B-III, C-IV, D-II
256. Two statements in respect of drug-enzyme interaction are given below
Statement I: Action of an enzyme can be blocked only when an inhibitor blocks the active site of the enzyme.
Statement II: An inhibitor can form a strong covalent bond with the enzyme.

Corporate Office : Aakash Tower, 8, Pusa Road, New Delhi-110005 [51]

For More Material Join: @JEEAdvanced_2024


For More Material Join: @JEEAdvanced_2024
Unique Practice Questions Chemistry

In the light of the above statements, choose the correct answer from the options given below
(1) Both Statement I and Statement II are true
(2) Both Statement I and Statement II are false
(3) Statement I is true but Statement II is false
(4) Statement I is false but Statement II is true
257. Given below are two statements: One is labelled as Assertion A and the other is labelled as
Reason R.
Assertion A: Thin layer chromatography is an adsorption chromatography.
Reason R: A thin layer of silica gel is spread over a glass plate of suitable size in thin layer chromatography
which acts as an adsorbent.
In the light of the above statements, choose the correct answer from the options given below
(1) Both A and R are true and R is the correct explanation of A.
(2) Both A and R are true but R is NOT the correct explanation of A
(3) A is true but R is false
(4) A is false but R is true
258. 100 mL of 5% (w/v) solution of NaCl in water was prepared in 250 mL beaker. Albumin from the egg was
poured into NaCl solution and stirred well. This resulted in a/an :
(1) Lyophilic sol (2) Lyophobic sol
(3) Emulsion (4) Precipitate
259. In metallurgy the term “gangue” is used for :
(1) Contamination of undesired earthy materials.
(2) Contamination of metals, other than desired metal.
(3) Minerals which are naturally occurring in pure form
(4) Magnetic impurities in an ore.
260. Which among the following pairs has only herbicides?
(1) Aldrin and Dieldrin (2) Sodium chlorate and Aldrin
(3) Sodium arsinate and Dieldrin (4) Sodium chlorate and sodium arsinite
261. Which of the following compounds is an example of hypnotic drug?
(1) Seldane (2) Amytal
(3) Aspartame (4) Prontosil
262. In a linear tetrapeptide (constituted with different amino acids) – (number of peptide bonds) is _____.
263. Which of the following is not a natural polymer?
(1) Protein (2) Starch
(3) Rubber (4) Rayon
264. Given below are two statements. One is labelled as Assertion A and the other is labelled as Reason R.
Assertion A : Amylose is insoluble in water.

Corporate Office : Aakash Tower, 8, Pusa Road, New Delhi-110005 [52]

For More Material Join: @JEEAdvanced_2024


For More Material Join: @JEEAdvanced_2024
Unique Practice Questions Chemistry

Reason R : Amylose is a long linear molecule with more than 200 glucose units. In the light of the above
statements, choose the correct answer from the options given below.
(1) Both A and R are correct and R is the correct explanation of A
(2) Both A and R are correct but R is NOT the correct explanation of A
(3) A is correct but R is not correct
(4) A is not correct but R is correct
265. A compound ‘X’ is a weak acid and it exhibits colour change at pH close to the equivalence point during
neutralization of NaOH with CH3COOH. Compound ‘X’ exists in ionized from in basic medium. The compound
‘X’ is
(1) Methyl orange (2) Methyl red
(3) Phenolphthalein (4) Eriochrome Black T

❑ ❑ ❑

Corporate Office : Aakash Tower, 8, Pusa Road, New Delhi-110005 [53]

For More Material Join: @JEEAdvanced_2024


For More Material Join: @JEEAdvanced_2024
Unique Practice Questions Chemistry

Corporate Office : Aakash Tower, 8, Pusa Road, New Delhi-110005


Ph.: 011-47623456

Unique Practice Questions (Chemistry)

Answers
1. (1) 26. (2) 51. (2)
2. (4) 27. (1) 52. (2)
3. (02.00) 28. (5) 53. (4)
4. (0.37) 29. (4) 54. (1)
5. (9) 30. (1) 55. (1)
6. (2) 31. (4) 56. (1)
7. (3.00) 32. (1) 57. (3)
8. (3) 33. (4) 58. (3)
9. (4) 34. (02.00) 59. (2)
10. (20.00) 35. (2) 60. (3)
11. (1) 36. (4) 61. (2)
12. (4) 37. (3) 62. (4)
13. (4) 38. (4) 63. (4)
14. (37.84) 39. (1) 64. (2)
15. (4) 40. (2) 65. (3)
16. (1) 41. (3) 66. (1)
17. (1) 42. (48) 67. (1)
18. (4) 43. (101) 68. (2)
19. (1) 44. (2) 69. (3)
20. (2) 45. (2) 70. (3)
21. (2) 46. (4) 71. (1)
22. (1) 47. (2) 72. (2)
23. (1) 48. (4) 73. (3)
24. (4) 49. (1) 74. (1)
25. (3) 50. (2) 75. (3)

Corporate Office : Aakash Tower, 8, Pusa Road, New Delhi-110005 [54]

For More Material Join: @JEEAdvanced_2024


For More Material Join: @JEEAdvanced_2024
Unique Practice Questions Chemistry

76. (1) 113. (2) 150. (3)


77. (2) 114. (1) 151. (2)
78. (2) 115. (3) 152. (4)
153. (2)
79. (1) 116. (2)
154. (4)
80. (4) 117. (1)
155. (2)
81. (2) 118. (1)
156. (1)
82. (1) 119. (2)
157. (2)
83. (4) 120. (1) 158. (2)
84. (4) 121. (1) 159. (1)
85. (4) 122. (3) 160. (4)
86. (1) 123. (1) 161. (3)
87. (1) 124. (4) 162. (3)
163. (1)
88. (2) 125. (1)
164. (03)
89. (1) 126. (1)
165. (04)
90. (3) 127. (4)
166. (2)
91. (2) 128. (3)
167. (1)
92. (2) 129. (4) 168. (3)
93. (4) 130. (4) 169. (2)
94. (4) 131. (2) 170. (01)
95. (2) 132. (4) 171. (2)
96. (1) 133. (4) 172. (1)
173. (1)
97. (4) 134. (4)
174. (3)
98. (1) 135. (1)
175. (4)
99. (3) 136. (2)
176. (2)
100. (2) 137. (2)
177. (02)
101. (2) 138. (3) 178. (3)
102. (4) 139. (2) 179. (4)
103. (3) 140. (4) 180. (2)
104. (1) 141. (3) 181. (1)
105. (4) 142. (4) 182. (1)
183. (3)
106. (4) 143. (2)
184. (2)
107. (1) 144. (2)
185. (09)
108. (1) 145. (1)
186. (3)
109. (2) 146. (1)
187. (4)
110. (4) 147. (3) 188. (3)
111. (1) 148. (1) 189. (4)
112. (3) 149. (2) 190. (1)

Corporate Office : Aakash Tower, 8, Pusa Road, New Delhi-110005 [55]

For More Material Join: @JEEAdvanced_2024


For More Material Join: @JEEAdvanced_2024
Unique Practice Questions Chemistry

191. (2) 216. (3) 241. (3)


192. (1) 217. (06) 242. (1)
193. (1) 218. (1) 243. (2)
195. (1) 219. (1) 244. (2)
195. (2) 220. (3) 245. (3)
196. (4) 221. (1) 246. (3)
197. (3) 222. (3) 247. (1)
198. (3) 223. (1) 248. (1)
199. (3) 224. (2) 249. (4)
200. (1) 225. (3) 250. (04.00)
201. (2) 226. (3) 251. (3)
202. (3) 227. (2) 252. (3)
203. (4) 228. (3) 253. (3)
204. (3) 229. (3) 254. (2)
205. (4) 230. (2) 255. (2)
206. (3) 231. (2) 256. (4)
207. (4) 232. (2) 257. (1)
208. (3) 233. (1) 258. (1)
209. (2) 234. (4) 259. (1)
210. (3) 235. (4) 260. (4)
211. (4) 236. (1) 261. (2)
212. (3) 237. (1) 262. (01)
213. (1) 238. (3) 263. (4)
214. (3) 239. (3) 264. (4)
215. (2) 240. (01.00) 265. (3)

Corporate Office : Aakash Tower, 8, Pusa Road, New Delhi-110005 [56]

For More Material Join: @JEEAdvanced_2024


For More Material Join: @JEEAdvanced_2024
Unique Practice Questions Chemistry

Solution

1. Answer (1) 30
for 250 ml, m mol of H2SO4 is required
42

 Weight of H2SO4 required

30
=  10−3  98 g = 0.3675 g  0.37 g
42

5. Answer (9)

Formed product is methyl orange and it is used


as an indicator in acid base titrations.
2. Answer (4)
Electron gain enthalpy is most negative for
chlorine followed by fluorine and bromine and Number of sp2 hybridized carbon are ‘9’.
least negative for iodine among given elements. 6. Answer (2)
So, F(–333 kJ mol–1) Fe(OH)3 is a positive sol. Its coagulation will be
Cl(–349 kJ mol–1) caused by the anion of the electrolyte.

Br(–325 kJ mol–1) The flocculation value is inversely proportional to


coagulation power or valency of the anion. The
I(–296 kJ mol–1)
correct order of flocculation value is
3. Answer (02.00)
The structure of chloramphenicol is K3 [Fe(CN)6] < K2CrO4 < KBr = KNO3 = AlCl3
7. Answer (3.00)
General structure of pencillin is

It contains 2 chiral centres.


The number of chiral centres in pencillin is 3.
4. Answer (0.37)
8. Answer (3)
Arsenic sulphide sol is negatively charged, so H+
The assertion is true, but the reason is false
brings about precipitation
Refer : [NCERT Page No. 132]
for 1 L, 30 m mol of HCl is required
Reactants must not get adsorbed so strongly
30
for 250 ml, m mol of HCl is required that they are immobilised and other reactants
4
are left with no space on the catalyst’s surface
for adsorption.

Corporate Office : Aakash Tower, 8, Pusa Road, New Delhi-110005 [57]

For More Material Join: @JEEAdvanced_2024


For More Material Join: @JEEAdvanced_2024
Unique Practice Questions Chemistry

9. Answer (4)  A = Aspartame, B = Saccharin


C = Sucralose, D = Alitame.
13. Answer (4)
Aniline and phenol form complex with Lewis
acid.

 Highest yield in Friedel Craft reaction among

Basicity of H3PO2 = 1
(Y)

10. Answer (20.00)


given species is for chlorobenzene i.e.
14. Answer (37.84)

Histamine :
120° = 3, 90° = 6,  180° = 1 Total = 10
Chemical formulae : C5H9N3

 % by mas of N

3  14
=  100
(5  12 + 1 9 + 3  14)

42  100
90° = 8, 180° = 2  Total = 10 = = 37.84%
111
Total number of 180°, 90° and 120° L-M-L bond
15. Answer (4)
angles = 10 + 10 = 20
All carbohydrates give, Molisch’s test
11. Answer (1)
- Barfoed test is specific for monosaccharide
Ce3+ (Z = 58) − [Xe]4f 1
- Biuret test is used for detecting the presence
2+ 7
Eu (Z = 63) − [Xe]4f of peptide bonds
12. Answer (4) 16. Answer (1)
• Ninhydrin test is specific for amino acids. BOD is the amount of oxygen required by

• Both aspartame and alitame contains amino bacteria to break down the organic matter

acids. present in a certain volume of sample of water.


17. Answer (1)
• Alitame is sulphur containing compound

thats why gives sodium nitroprusside test 2Pb(NO3 )2 ⎯⎯
→ 2PbO + 4NO2 + O2

• 
Saccharin also contains sulphur (NH4 )2 Cr2O7 ⎯⎯
→N2 + 4H2O + Cr2O3
• Sucralose contains chloro group thats why 
NH4NO2 ⎯⎯
→N2 + 2H2O
its Lassaigne extract gives white ppt with

AgNO3. 2NaN3 ⎯⎯
→ 2Na + 3N2

Corporate Office : Aakash Tower, 8, Pusa Road, New Delhi-110005 [58]

For More Material Join: @JEEAdvanced_2024


For More Material Join: @JEEAdvanced_2024
Unique Practice Questions Chemistry

18. Answer (4) 24. Answer (4)


Bunsen Burner and measuring cylinder is not From the thermal power plant CO2 is released
required for titration. which causes acid rain.
19. Answer (1) 25. Answer (3)
Respiratory ailments in animals may be due
UV
CF2Cl2 (g) ⎯⎯⎯
→ Cl(g) + CF2Cl(g) … (i)
to viral infections or bacterial pneumonia
etc; which can due to acid rain which
Cl(g) + O3 (g) ⎯⎯
→ ClO(g) + O2 (g)
Chlorine contains harmful chemicals acid rain is also
monoxide
harmless for trees.
… (ii) 26. Answer (2)
Drug Therapeutic action
ClO(g) + O(g) ⎯⎯
→ Cl(g) + O2 (g) … (iii)
(i) Ranitidine Antacid
Here, chlorine monoxide is formed in 2nd
(ii) Nardil Antidepressant
step so, statement (1) is not true.
(Phenelzine)
20. Answer (2)
(iii) Chloramphenicol Antibiotic
Aqueous NaHCO3 is used as the first aid.
NaHCO3 is basic and not harmful. (iv) Dimetane Antihistamine

21. Answer (2) 27. Answer (1)


An ionic micelle consists of an aggregate of
soap or detergents having a hydrophobic
part and a hydrophilic part. This aggregate
form a spherical shape with their
hydrocarbon chain pointing towards the
Ketose with seliwanoff’s reagent gives red centre of sphere.
colour. It is a specific test for ketose. 28. Answer (5)
22. Answer (1) The amino acids present in the given
For Unnilennium tripeptide Asp – Glu – Lys are
IUPAC symbol – Une
Atomic No. – 109
23. Answer (1)

and

It can react with Br2/water, ZnCl2/HCl and FeCl3.

Corporate Office : Aakash Tower, 8, Pusa Road, New Delhi-110005 [59]

For More Material Join: @JEEAdvanced_2024


For More Material Join: @JEEAdvanced_2024
Unique Practice Questions Chemistry

 Number of carbonyl groups present in Seldane is an anti-histamine drug that has


tripeptide = 2 + 2 + 1 = 5 inhibitory action on histamine receptor.
29. Answer (4) 34. Answer (02.00)
The structure of threonine is

30. Answer (1) No. of chiral centres present in it = 2


The bond which has greater potential energy 35. Answer (2)
(more negative) is considered more stable as it Noradrenaline is an organic chemical that
requires more energy to dissociate. functions in brain and body as hormone and

 A–B bond has most negative potential neurotransmitter.


Antidepressant drugs can be suggested for the
energy hence it is strongest bond and has
deficiency of noradrenaline.
maximum bond enthalpy.
36. Answer (4)
A–D is longest bond.
The amino acids present in the given tripeptide
31. Answer (4)
Ile–Arg–Pro are isoleucine, arginine and proline
Calcination : Heating of concentrated ore in the
absence (or limited supply of air). Mainly used
for carbonate ores.
Metal carbonate metal oxide + CO2
(causes Global
warming)

Roasting : Heating of concentrated ore in a


regular supply of air. Mainly used for sulphide
ores.
Metal sulphide + O2 ⎯→ Metal oxide + SO2
(causes acid rain)

32. Answer (1)


Number of chiral carbons present in the given
XeF4 + SbF5 → [XeF3 ]+ [SbF6 ]–
3 2 3
tripeptide is 4.
sp d sp d
37. Answer (3)
Disproportionation
H3PO2 ⎯⎯⎯⎯⎯⎯⎯⎯→ H3PO4 + PH3 Clark’s method is used to remove temporary
3 3 3
sp sp sp
hardness of water.
+
H + 38. Answer (4)
NH3 ⎯⎯⎯
→ NH4
3 3 Bronze contains 88 – 96% Cu and 4 – 12% Sn
sp sp

33. Answer (4) Cast iron is used to make wraugut iron

Corporate Office : Aakash Tower, 8, Pusa Road, New Delhi-110005 [60]

For More Material Join: @JEEAdvanced_2024


For More Material Join: @JEEAdvanced_2024
Unique Practice Questions Chemistry

German silver contains 50% Cu, 30% Zn and = 48 × 10–4


20% Ni 43. Answer (101)

Brass contains 70% Cu and 30% Zn. Unnilunium

39. Answer (1) IUPAC symbol = Unu


Rate = k[A]n Atomic no. (Z) = 101
log[Rate] = log k + n log [A] 44. Answer (2)
slope = n [n is order of the reaction] “-Helix is one of the most common ways in

 Correct sequence for the order of the which a polypeptide chain forms all possible
hydrogen bonds by twisting into a right handed
reaction is
screw (helix) with the –NH group of each amino
d>b>a>c
acid residue hydrogen bonded to the
40. Answer (2)
Ca(OH)2 – Used in white wash of an adjacent turn of the helix”
NaCl – Used for the preparation of 45. Answer (2)
washing soda (Na2CO3) Gun metal has composition of Cu, Zn, Sn

1 Cu – 87%
CaSO4  H2 O – Used for making casts of
2 Zn – 3 %
statues
Sn – 10%
CaCO3 – Used as an antacid 46. Answer (4)
41. Answer (3) During anaerobic degradation of vegetation CO2
Misch metal is an alloy consisting mainly of and CH4 are released which may lead to cause
lanthanoid metals. global warming and cancer.
42. Answer (48) 47. Answer (2)
Freundlich adsorption isotherm: Monomer Polymer
x Isoprene Natural rubber
= Kp1/n
m 2-Chloro-1, 3-butadiene Neoprene
x 1
 log = log K + log p Caprolactum Nylon 6
m n
Acrylonitrile Buna-N
1
= 2 and log K = 0.4771 48. Answer (4)
n
The amount of BOD in the water is a measure of
= log 3
the amount of organic material in the water, in
 K=3 terms of how much oxygen will be required to
x break it down biologically. Clean water would
= 3· p2
m have BOD value of less than 5 ppm, whereas
mass of gas adsorbed per gram of adsorbent highly polluted water would have a BOD value of
= 3 × (0.04)2 17 ppm or more.
Statement I is true and statement II is false.

Corporate Office : Aakash Tower, 8, Pusa Road, New Delhi-110005 [61]

For More Material Join: @JEEAdvanced_2024


For More Material Join: @JEEAdvanced_2024
Unique Practice Questions Chemistry

49. Answer (1) German silver contains Cu (50%), Zn (30%),


Valium Tranquilizer Ni (20%) respectively.
Morphine Analgesic 55. Answer (1)

Norethindrone Antifertility drug


Vitamin B12 Pernicious anaemia
(a)-(iv), (b)-(iii), (c)-(i), (d)-(ii)
50. Answer (2)

51. Answer (2)


Ce and Eu have stable oxidation state of +3. So,
Ce+4 acts as oxidizing agent to get reduced to +3 56. Answer (1)
and Eu2+ acts as reducing agent to get oxidized Kernite : Na2B4O7 . 4H2O (Boron)
to +3. Cassiterite : SnO2 (Tin)
52. Answer (2)
Calamine : ZnCO3 (Zinc)
Reducing smog is a mixture of smoke, fog and
Cryolite : Na3AlF6 (Fluorine)
sulphur dioxide. It does not involve O3 (allotrope
(a)-(ii), (b)-(i), (c)-(iv), (d)-(iii)
of oxygen) during its formation.
57. Answer (3)
The main component of the photochemical
Ozone in stratosphere (not in troposphere)
smog result from the action of sunlight on
protects us from U.V. radiation.
unsaturated hydrocarbons and nitrogen oxides.
No involvement of oxides of S. Ozone in troposphere generates photochemical
smog.
So, the answer should be, both statements false.
58. Answer (3)
53. Answer (4)
Vitamin K is helpful in delaying the blood clotting.
59. Answer (2)
Calgon is sodium hexametaphosphate, a
polymeric compound also called as Graham’s
salt.
Silicon is the 2nd most abundant element which
is absent in calgon.
enough to liberate CO2 with NaHCO3 solution.
60. Answer (3)
Answer (4)
54. Answer (1)

Corporate Office : Aakash Tower, 8, Pusa Road, New Delhi-110005 [62]

For More Material Join: @JEEAdvanced_2024


For More Material Join: @JEEAdvanced_2024
Unique Practice Questions Chemistry

there is a gradual decrease in the size of M3+


ions across the series. This may be referred to
as the actinoid contraction.
65. Answer (3)
Ordinary filter paper cannot stop the flow
colloidal particles.
An ultra - filter paper is required.
(a) → (ii), (b) → (i), (c) → (iii) 66. Answer (1)
61. Answer (2) Lassaigne’s test is used in qualitative detection
Vitamins which are soluble in fat and oils but of N, S, P and halogens.
insoluble in water are fat soluble vitamins, which (a) (iii)
are stored in our body relatively for longer time. Cu(II) oxide is used for the estimation of carbon.
e.g. Vitamin A and Vitamin D (b) (i)
Thiamine (Vit B1) and Ascorbic acid (Vit C) are Silver nitrate is used for detecting halogens in
water soluble. organic compounds (Carius Method)
62. Answer (4)
(c) (iv)
Process Application
Sodium fusion extract when treated with acetic
Haber’s process NH3 synthesis
acid and lead acetate gives black precipitate if
Ostwald’s process HNO3 synthesis sulphur is present.

Contact process H2SO4 synthesis S2– + Pb 2 + → PbS


( black )
Hall Heroults process Aluminium
(d) (ii)
extraction
67. Answer (1)
(a)-(iii), (b)-(i), (c)-(iv), (d)-(ii) Carbon dioxide, water vapour, methane are
63. Answer (4) greenhouse gases.
Photochemical smog occurs in warm, dry and 68. Answer (2)
sunny climate. It is also called oxidising smog. A colloidal system consisting of a gas dispersed
The main components of photochemical smog in a solid is called a ‘foam’.
are ozone, nitric oxide, acrolein, formaldehyde 69. Answer (3)
and PAN Retardation factor (Rf) is the ratio of distance
64. Answer (2) moved by the substance from the base line to
Size of Bk3+ is 98 pm the distance moved by the solvent from the base
Size of Np3+ is 101 pm line. So, it is dimensionless. The distance moved

So size of Np3+ is more than Bk3+ ion. by the substance is due to adsorption of the
substance on the stationary phase. It does not
depend on the nature of solvent. But the
distance moved by the solvent will change with

Corporate Office : Aakash Tower, 8, Pusa Road, New Delhi-110005 [63]

For More Material Join: @JEEAdvanced_2024


For More Material Join: @JEEAdvanced_2024
Unique Practice Questions Chemistry

the nature of solvent. Therefore, Rf will vary with 1


CaSO4 H2O - Plaster of Paris
the change in solvent. 2

So, both the statements are false. Ca(OCl)2 - Bleaching agent


70. Answer (3) CaO is used in cement
Heavy water (D2O) is obtained as a by-product CaCO3 is used as an Antacid
in fertilizer industry. It is used as a moderator in 76. Answer (1)
nuclear reactor and for the study of reaction Chlorophyll - Mg
mechanism. Its dielectric constant is lower than
Vitamin - B12 - Co
that of H2O.
Anticancer drug - Pt
71. Answer (1)
77. Answer (2)
Allosteric inhibitor changes the enzymes active
Antacid - Cimetidine
site and they do not compete with the enzymes
Artificial Sweetener - Alitame
active site. They bind to the allosteric site.
Antifertility - Novestrol
Competitive inhibitor binds to the enzymes
active site. Tranquilizers - Valium

72. Answer (2) 78. Answer (2)

Invertase (a) Deacon's process - CuCl2


C12H22O11 + H2O ⎯⎯⎯⎯→ C6H12O6 + C6H12O6
Sucrose Glucose Fructose (b) Contact process - V2O5
Zymase
C6H12O6 ⎯⎯⎯⎯→ 2C2H5OH + 2CO2 (c) Cracking of - ZSM-5
Glucose
hydrocarbons
 Enzyme A is Invertase
(d) Hydrogenation of - Particles 'Ni'
Enzyme B is Zymase
vegetable oils
73. Answer (3)
79. Answer (1)
(a) Sucralose → (ii) Artificial sweetener
Cryolite is used to lower M.P. of Alumina.
(b) Glyceryl ester of stearic acid → (i) Synthetic 80. Answer (4)
NCERT, Page No. – 415, 417 – Part-II, Class-XI
detergent 81. Answer (2)
(c) Sodium benzoate → (iv) food preservative Deficiency of vitamin K causes increase in blood

(d) Bithionol → (iii) Antiseptic clotting time.

74. Answer (1) 82. Answer (1)

Nutrient enriched water bodies supports dense (a) Antifertility drug — Norethindrone

plant population. This is because of the excess (b) Antibiotic — Salvarsan


usage of fertilisers and detergents. This process (c) Tranquilizer — Meprobamate
is known as eutrophication. (d) Artificial Sweetener — Alitame
75. Answer (3) (a)-(iii), (b)-(iv), (c)-(i), (d)-(ii)
83. Answer (4)

Corporate Office : Aakash Tower, 8, Pusa Road, New Delhi-110005 [64]

For More Material Join: @JEEAdvanced_2024


For More Material Join: @JEEAdvanced_2024
Unique Practice Questions Chemistry

Element Refining method Enzymes are mostly proteins. They function as


(a) Mercury Distillation refining catalysts in biochemical reactions by lowering
(b) Copper Electrolytic refining the energy of activation. They are highly specific

(c) Silicon Zone refining w.r.t. temperature and pH in their action.


89. Answer (1)
(d) Nickel Vapour phase refining
Nx(at t) = N0e–5t
So, (a)-(ii), (b)-(iii), (c)-(iv), (d)-(i).
84. Answer (4) Ny(at t) = N0e–t
Be – used in making windows of X-ray tube Nx 1
= 2
= e –4 t
Mg – incendiary bombs and signals Ny e
Ca – extraction of metals  4t = 2
Ra – treatment of cancer 2  1
 t= = 
85. Answer (4) 4   2 
The diameter of the dispersed particles is not 90. Answer (3)
much smaller than the wavelength of the light
Vitamin B1 — Thiamine
used; and
Vitamin B6 — Pyridoxine
The refractive indices of the dispersed phase
91. Answer (2)
and the dispersion medium differ greatly in
Ba — [Xe]6s2
magnitude.
86. Answer (1) Ca — Calcium oxalate is insoluble (sparingly
soluble) in water
Li — LiCl is soluble in organic solvents like
pyridine
Na — NaOH is a very strong monoacidic base
92. Answer (2)
Froth floatation is a method of concentration and
it was discovered by a washer women.
93. Answer (4)
[SiCl6]2– is not known. The main reasons are (i)
six large chloride ion cannot be accommodated
around Si4+ due to limitation of its size and (ii)
interaction between lone pair of chloride ion and
Si4+ is not very strong.
87. Answer (1) 94. Answer (4)
Amylose is a unbranched chain with 200–1000 Surfactant form micelles by pointing the
-D-(+)-glucose units held together by C1-C4 hydrophobic part towards the centre of sphere.
glycosidic linkage. 95. Answer (2)
88. Answer (2) Given polymer is novolac

Corporate Office : Aakash Tower, 8, Pusa Road, New Delhi-110005 [65]

For More Material Join: @JEEAdvanced_2024


For More Material Join: @JEEAdvanced_2024
Unique Practice Questions Chemistry

96. Answer (1) [HN – CH2 – CO – NH – (CH2 )5 – CO] n


Globular proteins are usually soluble in water. Nylon − 2 − nylon − 6

Insulin and albumin are the common examples 101. Answer (2)
of globular proteins. Cr has tendency to form hydride easily
97. Answer (4) Elements of group – 7 and 8 do not form
Chlorofluoro carbons breakdown by radiation in hydrides
the UV region and not in the visible region to give 102. Answer (4)
chlorine free radical which reacts with NaOH – Basic
stratospheric ozone. Ca(OH)2 – Basic
Be(OH)2 – Amphoteric
Atmospheric ozone reacts with NO to give NO2 Al(OH)3 – Amphoteric
and O2 and not N2 and O2 B(OH)3 – Acidic
103. Answer (3)

Both the given statements are false. The given compound (A) is Thymine. It always

98. Answer (1) bind with adenine in DNA.

Cheese – Liquid dispersed in solid


Pumice stone – Gas dispersed in solid
Hair cream – Liquid dispersed in liquid
Cloud – Liquid dispersed in gas

 Correct answer is

(a) - (iv), (b) - (iii), (c) - (i), (d) - (ii)


99. Answer (3)
104. Answer (1)
Concentration of Ag ore – Leaching with
The lack of oxygen kills all other forms of aquatic
dil. NaCN solution
life such as fish and plants. Fertilizers contain
Blast furnace – Pig iron phosphates as additives. The addition of
Blister copper – Reverberatory phosphates in water enhances algae growth.
furnace Such profuse growth of algae, covers the water
Froth floatation method – Sulphide ores surface and reduces the oxygen concentration
 Correct match is in water. This leads to anaerobic conditions,

(a) - (iii), (b) - (ii), (c) - (i), (d) - (iv) commonly with accumulation of abnoxious

100. Answer (2) decay and animal death. Thus, bloom-infested

Glycine and aminocaproic acid undergo water inhibits the growth of other living

condensation polymerisation to form Nylon-2- organisms in the water body. This process in

nylon-6, a biodegradable polymer. which nutrient enriched water bodies support a


dense plant population, which kills animal life by
H2N – CH2 – COOH + H2N – (CH2 )5 – COOH →

Corporate Office : Aakash Tower, 8, Pusa Road, New Delhi-110005 [66]

For More Material Join: @JEEAdvanced_2024


For More Material Join: @JEEAdvanced_2024
Unique Practice Questions Chemistry

depriving it of oxygen and results in subsequent NaCl. The resin exchanges Na+ ions with Ca2+
loss of biodiversity is known as Eutrophication. and Mg2+ ions present in hard water to make the
105. Answer (4) water soft. Here R is resin anion.
Barfoed’s is a chemical test used to detect 2RNa(s) + M2+(aq) → R2M(s) + 2Na+(aq)
presence of monosaccharides from
The resin can be regenerated by adding
disaccharides.
aqueous NaCl solution.
106. Answer (4)
Pure de-mineralised (de-ionized) water free
Correct match are
from all soluble mineral salts is obtained by
(a) Furacin – Antiseptic
passing water successively through a cation
(b) Arsphenamine – Antibiotic exchange (in the H+ form) and an anion -
(c) Dimetone – Synthetic exchange (in the OH– form) resins:
antihistamine
2RH(s) + M2+ (aq) MR2 (s) + 2H+ (aq)
(d) Valium – Tranquilizers
In this cation exchange process, H+ exchanges
107. Answer (1)
for Na+, Ca2+, Mg2+ and other cations present in
Penicillin is a bactericidal antibiotics.
water. This process results in proton release and
108. Answer (1)
thus makes the water acidic.
Silica converts iron oxide into iron silicate.
111. Answer (4)
FeO + SiO2 ⎯⎯
→ FeSiO3 Gly-Glu-Asp-Tyr
109. Answer (2)
(a) CO ......... (iii) Haemoglobin
(b) SO2 ......... (iv) Stiffness to flower buds
(c) Polychlorinated ...(i) Carcinogenic
biphenyls
(d) Oxides of nitrogen ...(ii) Metabolized At pH = 12.5
by pyrus plants —COOH, and phenol will lose its proton
110. Answer (1)
Synthetic resin method is most suitable for
preparing deionize water.
Synthetic resins method: Nowadays hard
water is softened by using synthetic cation
112. Answer (3)
exchangers. This method is more efficient than
Given compound is an example of tranquilizer.
zeolite process. Cation exchange resins contain
113. Answer (2)
large organic molecule with - SO3H group and
Sucrose is a disaccharides and a non-reducing
are water insoluble. Ion exchange resin
sugar because it does not contain free
(RSO3H) is changed to RNa by treating it with
hemiacetal linkage.

Corporate Office : Aakash Tower, 8, Pusa Road, New Delhi-110005 [67]

For More Material Join: @JEEAdvanced_2024


For More Material Join: @JEEAdvanced_2024
Unique Practice Questions Chemistry

Sucrose involves glycosidic linkage between C 1 • The process of producing syn - gas is called
of –D–glucose and C2 of –D– fructose. ‘coal gasification’ — statement 1 is true
114. Answer (1) • Composition of syn - gas is CO : H2 as 1 : 1
Photochemical smog causes cracking of rubber. 121. Answer (1)
It contains ozone, nitric oxide, acrolein, When the polypeptide chains run parallel and
formaldehyde and peroxyacetyl nitrate. It are held together by hydrogen and disulphide
contains high concentration of oxidizing agents bonds, then fibre-like structure is formed. Such
and is, therefore, called as oxidizing smog. proteins are generally insoluble in water. Some
115. Answer (3) common examples are keratin (present in hair,
(a) Diamagnetism — (iii) NaCl wool, silk) and myosin (present in muscles), etc.
(b) Ferrimagnetism — (iv) Fe3O4 This structure results when the chains of
(c) Paramagnetism — (ii) O2 polypeptides coil around to give a spherical

(d) Antiferromagnetism — (i) MnO shape. These are usually soluble in water.

116. Answer (2) Insulin and albumins are the common examples

Compound (B) is Valium and compound (C) is of globular proteins.

serotonin. They are used as tranquilizers. 122. Answer (3)

117. Answer (1) Ammonium salts are also formed and can be
seen as an atmospheric haze (aerosol of fine
Seliwanoff’s test → Resorcinol dissolved in conc
particles). Aerosol particles of oxides or
HCl.
ammonium salts in rain drops result in wet-
All other test use copper based reagent.
deposition. SO2 is also absorbed directly on both
118. Answer (1)
solid and liquid ground surfaces and is thus
Lyophilic sols are more stable because they are deposited as dry-deposition.
solvated in solution. They are also called as 123. Answer (1)
solvent loving. Lesser the value of BOD, cleaner will be the
119. Answer (2) water sample
Biuret ligand is 124. Answer (4)
In reaction (i), Au (sol) is formed by reduction of
AuCl3, so the chemical method of preparation is
“Reduction”.

It forms complexes like In reaction (ii), As2S3 (sol) is formed by double


decomposition, so the chemical method of
preparation is “Double decomposition”.
In reaction (iii), S (sol) is formed by oxidation of
H2S, so the chemical method of preparation is
“Oxidation”.
120. Answer (1)

Corporate Office : Aakash Tower, 8, Pusa Road, New Delhi-110005 [68]

For More Material Join: @JEEAdvanced_2024


For More Material Join: @JEEAdvanced_2024
Unique Practice Questions Chemistry

In reaction (iv), Fe(OH)3(sol) is formed by


hydrolysis of FeCl3 so the chemical method of
preparation is “Hydrolysis”.
125. Answer (2)
• Chloroxylenol is dettol contain phenolic
group so give FeCl3 test
129. Answer (4)
• Norethindrone has double bond so will give Density of dichloromethane is greater than
Baeyer's reagent test water. So, DCM would be the lower layer and
• Sulphapyridine has – NH2 group it give water will form the upper layer in the separating
carbyl amine test funnel.
• Penicillin has – COOH group so will respond 130. Answer (4)
to NaHCO3 test

Asn 

Ser 

Asn – Ser

126. Answer (1)


Down the group as the charge density 131. Answer (2)
Lysine (amino - acid) reacts with ninhydrin to
decreases so chances of formation of hydrate
give a coloured product (blue purple)
decreases.
– In furfural test (to distinguish between glucose
So, Ba(NO3)2 does not crystallise with water
and fructose) dilute sugar solution is added to 1-
molecules. naphthol (in alcohol) and conc. HCl.
127. Answer (4) – Benzyl alcohol is oxidised to aldedydes using
Alumina is an adsorbent (stationary phase) ceric ammonium nitrate
Benzaldehyde is adsorbate. Styrene is converted to
Acetonitrile is mobile phase.
128. Answer (3)

Corporate Office : Aakash Tower, 8, Pusa Road, New Delhi-110005 [69]

For More Material Join: @JEEAdvanced_2024


For More Material Join: @JEEAdvanced_2024
Unique Practice Questions Chemistry

136. Answer (2)


Allosteric effect: Molecule bind to a site other
than the active site of enzyme
using KMnO4
Competitive
132. Answer (4)
inhibitor: Molecule bind to the acitve
site

Receptor: Molecule crucial for


communication in the body.

Poison: Molecule binding to the


enzyme covalently
137. Answer (2)
Phenol or unsaturated hydrocarbon (alkene or
alkyne) decolourised bromine water solution.

133. Answer (4) C2H5MgBr will react with carbonyl carbon or

acidic hydrogen.

reacts with C2H5MgBr and

also decolourized bromine water solution


134. Answer (4)
138. Answer (3)
Wilkinson catalyst is [Rh(PPh)3Cl]
dE
Chlorophyll contains Mg. r H = −nFE + nFT
dT
Vitamin B12 contains Co. 139. Answer (2)
Carbonic anhydrase contains Zn. K+ ions act as carriers for nerve signals, are
135. Answer (1) activators for many enzymes and participate in
the oxidation of glucose to form ATP.
140. Answer (4)
Histidine (in strongly acidic solution)

(P)

(Q) AsP – –CH2–COOH – Ester test (A)

(R) Ser – –CH2–OH – Ester test (A)

(S) Lys – –(CH2)4–NH2 – Carbylamine test (B)

Corporate Office : Aakash Tower, 8, Pusa Road, New Delhi-110005 [70]

For More Material Join: @JEEAdvanced_2024


For More Material Join: @JEEAdvanced_2024
Unique Practice Questions Chemistry

141. Answer (3) 1


A(g) B(g) + C(g)
2
t=0 pi − −
pi 
t=t pi − pi  pi 
( eq. m conditions) 2

pi 
pi − pi  + pi  +
∴ P (equilibrium pressure) = 2
142. Answer (4)
 
Polysubstitution is a major drawback in Friedel pi  1 + 
Craft’s alkylation =  2

143. Answer (2)


p
Seliwanoff’s test is used to distinguish aldose  pi =
 
and ketose 1 + 2 
 
144. Answer (2)
1
  2
 pi 2   pi 
1 3
p2 2
KP =   1
= 
pi (1 − ) 1 1
  2
22
 1 + 2  (1 − )
 
More stable product due to conjugation .
145. Answer (1) 1 3
p2 2
Ceric ammonium nitrate test is given by alcohol. = 1
Only serine(ser) contain –OH group. (2 +  ) 2 (1 −  )
146. Answer (1)
Activation enthalpy to form C is 5 kJ more than Hence the correct

that to form D. 150. Answer (3)


147. Answer (3) Oil in water emulsions can sometimes separate
Quartz, tridymite and cristobalite are crystalline
into two layers on standing.
forms of silica.
The most relevant example for the above case
Kieselguhr is an amorphous form of silica.
is milk, which can separate into two layers on
148. Answer (1)
In column chromatograph a solid adsorbent is standing for a longer time. Therefore, statement

packed in a column and a solution containing (I) is correct.


number of solute particles is allowed to flow On adding excess of electrolyte, coagulation
down the column. The solute molecules get occurs and emulsion is further destabilised.
adsorbed on the surface of adsorbent. So, it is
Therefore, statement (II) is incorrect.
differential adsorption of the substances on the
151. Answer (2)
solid phase.
149. Answer (2)

Corporate Office : Aakash Tower, 8, Pusa Road, New Delhi-110005 [71]

For More Material Join: @JEEAdvanced_2024


For More Material Join: @JEEAdvanced_2024
Unique Practice Questions Chemistry

On boiling with dil. H2SO4 at 393 K under 2-3


atm, ‘X’ forms glucose, which given gluconic
acid on treatment with bromine water.
154. Answer (4)
Penicillin G is a narrow spectrum antibiotic.
152. Answer (4) (Based on fact)
Polyesters are formed by condensation reaction 155. Answer (2)
between alcohols and carboxylic acid. P+3 is not present is enamel of teeth.
Poly-β-hydroxybutyrate-co-β-hydroxy valerate Th compound present is [3Ca3(PO4)2·CaF2]
(PHBV) is a polymer obtained by condensation
Which contains Ca+2, P+5 & F–
reaction of 3-hydroxybutanoic acid with
156. Answer (1)
3-hydroxypentanoic acid.
Ores Formula

(A) Sphalerite (IV) ZnS

(B) Calamine (III) ZnCO3

(C) Galena (II) PbS

(D) Siderite (I) FeCO3

Hence, the most appropriate option is (A).


157. Answer (2)
Melting points of the given metals
Hence, PHBV is a polyester. Hg : –38.83° C
153. Answer (2) Ag : 961.8° C
Cellulose contains β-glycosidic linkages only. Ga : 29.76° C

Structure of cellulose Cs : 28.44° C


∴ Metal having highest melting point is Ag.

Corporate Office : Aakash Tower, 8, Pusa Road, New Delhi-110005 [72]

For More Material Join: @JEEAdvanced_2024


For More Material Join: @JEEAdvanced_2024
Unique Practice Questions Chemistry

158. Answer (2) Sr2+ Crimson red

Hall-Herault process is used for the extraction of Ca2+ Brick red


aluminium by electrolysis molten Al2O3 Ba2+ Green
2Al2O3 + 3C → 4Al + 3CO2 164. Answer (03)

159. Answer (1) Baryte BaSO4


Molecular hydrogen is produced as a byproduct Galena PbS
in the industrial production of NaOH by Zinc blende ZnS
electrolysis of aq NaCl solution
Copper pyrites CuFeS2
NaCl → Na+ + Cl–
Of the given minerals, only 3 are sulphide
based.
H2O H+ + OH–
165. Answer (04)
Cathode : 2H2O + 2e → H2 + 2OH–
The given pentapeptide is

Anode : 2Cl → Cl2 + 2e ALA – GLY – LEU – ALA – VAL
It has 4 peptide linkages.
NaOH is crystallised from the remaining part of
166. Answer (2)
electrolyte.
For Tyndall effect, the diameter of the dispersed
160. Answer (4)
particles is not much smaller than the
Among the given gases, the green house
wavelength of the light used.
gases which are responsible for heating the
167. Answer (1)
atmosphere are CH4, water vapour and ozone.
Eutrophication is the process in which nutrient
Nitrogen is not a green house gas.
enriched water bodies support a dense plant
161. Answer (3)
population, which kills animal life by depriving it
Nylon 6, 6 is a condensation polymer of
of oxygen and results in subsequent loss of
hexamethylene diamine and adipic acid
biodiversity.
Dacron is a condensation polymer of
168. answer (3)
terephthalic acid and ethylene glycol.
The little amount of using soap while washing
Buna-N is an addition polymer of 1, 3-butadiene clothes, does not serve the purpose of cleaning
and acrylonitrile of clothes because the micelles are not formed
Silicone is a condensation polymer of dialkyl due to the low concentration of soap as it is
silanediol. below CMC.
162. Answer (3) 169. Answer (2)
The given structure is that of cimetidine which Alitame is an artificial sweetner.
is well known antacid. 170. Answer (01)
163. Answer (1)
1  −d[A]  1  −d[B]  1  −d[C] 
Cupric salts give green flame with blue centre.  =  =  
Rate = 1  dt  r2  dt  r3  dt 
r
The colour of other salts are

Corporate Office : Aakash Tower, 8, Pusa Road, New Delhi-110005 [73]

For More Material Join: @JEEAdvanced_2024


For More Material Join: @JEEAdvanced_2024
Unique Practice Questions Chemistry

1  d[D]  Urease → Soyabea


=
r4  dt  n

d[D] r4  −d[B] 
Pepsin → Stomach
= 
dt r2  dt  Hence, A-II, B-III, C-IV, D-I
172. Answer (1)
r4 3
= Copper is refined using an electrolytic method.
r2 2
Anodes are of impure copper and pure copper
−d[B] r2  −d[A]  r strips are taken as cathode
=   2 =2
dt r1  dt  r1
Impurities from the blister copper deposit as
anode mud which contains antimony, selenium,
r2 = 2r1
tellurium, silver, gold and platinum.
r4 = 1.5r2 = 3r1 Hence both statements are true

d[C] 173. Answer (1)


dt = 1 m.mol dm–3 sec–1 2Pb(NO3 )2 ⎯⎯⎯
673 K
→ 4NO2 + 2PbO + O2
A

d[D]
NO2 ⎯⎯⎯⎯
Dimerise
→N2O4
dt = 9 m.mol dm–3 sec–1 A B

d[D] r4 d[C] r
=   4 =9
dt r 3 dt r3

r4 = 9r3 = 3r1

 r1 = 3r3 Hence no bridged oxygen atom is present in


N2O4.
3r3A + 6r3B → r3C + 9r3D
174. Answer (3)
1
9 Highly polluted water could have a BOD value of
∴ rate of reaction = 9 m.mol dm–3 sec–1
17 ppm or more.
= 1 m.mol The amount of oxygen required by bacteria to
dm–3 sec–1 break down the organic matter present in a
171. Answer (2) certain volume of a sample of water is called
Enzyme Source Biochemical Oxygen demand (BOD).

Zymase → Yeast Hence A is correct but R is not correct.


175. Answer (4)
Diastas → Malt
Since, both benzoic acid and naphthalene will
e
dissolve in benzene. Hence assertion is wrong.

Corporate Office : Aakash Tower, 8, Pusa Road, New Delhi-110005 [74]

For More Material Join: @JEEAdvanced_2024


For More Material Join: @JEEAdvanced_2024
Unique Practice Questions Chemistry

Benzoic acid is almost insoluble in cold water but at any specified temperature, is given by the
soluble in hot water. Hence Reason is true equation
176. Answer (2) G = H − TS
Dettol is a mixture of chloroxylenol and terpineol.
where H = enthalpy change and
Chloroxylenol has 6 electrons and terpineol has
S = entropy change
2 electrons.
According to the ellingham diagram, any metal
Hence 2 is terpineol.
177. Answer (02) oxide with higher G° has a tendency of getting

Since dipeptides and free amino acids do not reduced by the metal whose metal oxide has

give biuret test. Hence glycine and glycylalanine lower value of G°.

do not give this test. Therefore, Statement I is incorrect but


178. Answer (3) Statement II is correct.

We know that 180. Answer (2)

In summer season nitrogen dioxide and


2.303RT  1 
E = Eo − log  
nF  (oxidized form)  methane react with chlorine monoxide and
chlorine atoms forming, Chlorine sinks,
−2.303 R  1 
Slope = log   preventing much ozone depletion, whereas in
nF  (oxidized form) 
winter, special types of clouds called polar
stratospheric clouds are formed over Antarctica.
These polar stratospheric clouds provide
E surface on which chlorine nitrate formed gets
hydrolysed to form hypochlorous acid.

T ClO(g) + NO2 (g) → ClONO2 (g)

 E  ClONO2(g) + H2O(g) → HOCl(g) + HNO3(g)


Higher the value of slope   , more the
 T P 181. Answer (1)
deviation between E and E°. Thus, half cell with NaSCN + 2Na → NaCN + Na2S
 E  182. Answer (1)
least value of   can be used as reference
 T P
(A) Buna-S – Copolymer
electrode.
(B) Nylon-6,6 – Condensation polymer
179. Answer (4)
(C) Nylon-6,6 – Fibre
Ellingham diagram is plot of G vs T.
(D) Terylene – Thermoplastic
The criterion for the feasibility of a thermal
(E) Buna-N – Copolymer
reduction is that at a given temperature Gibbs
energy change of a reaction must be negative. (A) A, B and C are correct.
183. Answer (3)
The change in Gibbs energy, G for any process

Corporate Office : Aakash Tower, 8, Pusa Road, New Delhi-110005 [75]

For More Material Join: @JEEAdvanced_2024


For More Material Join: @JEEAdvanced_2024
Unique Practice Questions Chemistry

Histamine stimulates the secretion of pepsin and Vitamin B12 is water soluble and not excreted
hydrochloric acid in the stomach. easily.
190. Answer (1)

Correct match of List-I and List-II is:

(A) Lyophilic colloid (II) Protective colloid


184. Answer (2)
(B) Emulsion (I) Liquid-liquid
Brown ring test
colloid
NO3− + 3Fe +2
+ 4H+ → NO+ 3Fe+3+2H2O (C) Positively charged (IV) FeCl3 + hot water

[Fe(H2O)6]2+ NO → [Fe(H2O)5NO]2+ H2O colloid (It forms a

Brown ring positively


185. Answer (09) charged sol of
Nucleotide formed by Uracil, the base present in Fe(OH)3)
RNA, is
(D) Negatively charged (III) FeCl3 + NaOH

colloid (Negatively

charged

colloid is formed

due

The number of oxygen = 9 to adsorption of

186. Answer (3) OH–

(A) Invertase → Cane sugar into glucose ions on Fe(OH)3)

(B) Zymase → Glucose into ethanol 191. Answer (2)

(C) Diastase → Starch into maltose List-I List-II

(D) Maltase → Maltose into glucose (A) Concentration of (IV) NaCN

187. Answer (4) Gold ore


Beryllium does not give flame test because of its (B) Leaching of alumina (II) NaOH
small size and high ionization energy the energy
(C) Froth stabiliser (I) Aniline
of flame is not sufficient to excite the electrons
(Aniline and
to higher energy level
cresols
188. Answer (3)
Highly polluted water should have BOD value of are used as froth

17 ppm or more stabilisers in froth


189. Answer (4) floatation
process)

Corporate Office : Aakash Tower, 8, Pusa Road, New Delhi-110005 [76]

For More Material Join: @JEEAdvanced_2024


For More Material Join: @JEEAdvanced_2024
Unique Practice Questions Chemistry

(D) Blister copper (III) SO2 (During self


reduction
process used in → Cationic detergent

the formation of
blister copper
SO2 gas is
(Hair conditioner)
evolved)

Hence (2) is most appropriate option.


192. Answer (1) (B)

(I) Classical smog occurs in cool humid


→ Toothpaste
climate. It is a reducing mixture of smoke, fog
(Anionic detergent)
and sulphur dioxide. This is a correct statement.
(C) C17H35COO–Na++Na2CO3+Rosinate
(II) This statement is also based on fact and is
a correct statement. → Laundry soap
193. Answer (1) (D) CH3(CH2)16COO(CH2CH2O)CH2CH2OH
The diagram is option (A) clearly represents

separating funnel which is used to separate two
Dishwashing powder
immiscible liquids.
196. Answer (4)
194. Answer (1)
Leaching of gold with cyanide ion is done in
List-I List-II presence of air/O2 leading to cyano complex
(Polymer) (Used in) [Au(CN)2]– where Au is in +1 oxidation state.
(A) Bakelite (II) Electrical switches 4Au(s) + 8CN–(aq) + 2H2O(aq) + O2(g)

(B) Glyptal (III) Paints and ⎯⎯→ 4  Au ( CN)2  + 4OH–
aq
Lacqures

2  Au ( CN)2  + Zn ( s )
(C) PVC (IV) Water pipes ( aq)
2–
(D) Polystyrene (I) Radio and ⎯⎯→  Zn ( CN )4  + 2Au ( s )
( aq)
television
Zinc is oxidised from (0) to +2 oxidation state
Cabinets
during displacement reaction carried out for gold
Therefore, the correct option is (1). extraction.
195. Answer (2) 197. Answer (3)
+2 +3 +4
Eu 4f7 4f6
Tm 4f13 4f12
Sm 4f6 4f5

Corporate Office : Aakash Tower, 8, Pusa Road, New Delhi-110005 [77]

For More Material Join: @JEEAdvanced_2024


For More Material Join: @JEEAdvanced_2024
Unique Practice Questions Chemistry

Tb 4f9 4f8 4f7 The melting and boiling point of magnesium are
Yb 4f14 4f13 lower than those of aluminium but it is not the

Dy 4f10 4f9 correct reason.


202. Answer (3)
Hence, the pair Tb+4 Yb+2 have half filled and
completely filled f subshells respectively. Eu+2 is 4f7
198. Answer (3) Yb+2 is 4f14
Buna-N is formed by copolymerisation of 1-3- but Eu+2 is more stable than Yb+2 because
butadiene and acrylonitrile
E  E
Eu|Eu+2 Yb|Yb+2
CN
Copolymerisation 203. Answer (4)
nCH2 = CH – CH = CH 2 + CH2 = CH
S2– + [Fe(CN)5NO)]2– → [Fe(CN)5(NOS)]4–
CN

CH2 – CH = CH – CH 2 – CH2 – CH n Purple


204. Answer (3)
199. Answer (3) The function of mobile phase is to carry the
Maltose is composed of two -D-glucose units in components present on TLC.
which C1 of one glucose unit and C4 of second 205. Answer (4)
glucose unit are linked. Sodium arsenite is a herbicide.
6 6 206. Answer (3)
CH2OH CH2OH
5 (A) Negatively charged sol CdS sol
5 O
H H H H
H H (B) Macromolecular colloid Starch
4 1 4 1
OH H O OH H
HO OH (C) Positively charged sol Fe2O3.xH2O
3 2 3 2
H H (D) Cheese A gel
OH OH
207. Answer (4)
200. answer (1)
Antipyretic – Reduces fever FeO + SiO2 → FeSiO3

Analgesic – Reduces pain Gangue Slag


208. Answer (3)
Tranquilizer – Reduces stress
Antacid – Reduces Acidity (stomach)
201. Answer (2)

Magnesium can reduce Al2O3 at a temperature


below 1350°C while above 1350°C aluminium The cis-polyisoprene molecule consists of
can reduce MgO because below 1350°C G of various chains held together by weak van der
MgO (formation) is more negative and above Waals interactions and has a coiled structure.
1350°C G of Al2O3 (formation) is more Hence assertion is true but reason is false.
negative. 209. Answer (2)

Corporate Office : Aakash Tower, 8, Pusa Road, New Delhi-110005 [78]

For More Material Join: @JEEAdvanced_2024


For More Material Join: @JEEAdvanced_2024
Unique Practice Questions Chemistry

Histamine – does not contain sulphur.


+
C12H22O11 + H2O ⎯⎯⎯
H
→ C6H12O6 + C6H12O6
Sucrose D-( + )-Glu cos e D-( − )-Fructose
(A) (B)

CHO COOH
  217. Answer (06)
(CHOH)4 ⎯⎯⎯⎯⎯
nitric acid
→ (CHOH)4
 
CH2OH COOH FeCr2O4 + Na2CO3 + O2 ⎯⎯→ Fe2O3 + CO2
glucose
saccharic acid + Na2CrO4
(A)
D-(–)-Fructose is a laevorotatory compound.
+
210. Answer (3) Na2CrO4 + H+ ⎯⎯→ Cr2O7−2 + H2O + Na
Tegamet (cimetidine) is

218. Answer (1)

Siderite → FeCO3
211. Answer (4)
Malachite → CuCO3. Cu(OH)2
Presence of equal and similar charges on
colloidal particle provides stability to the colloidal Sphalerite → ZnS

solution. Calamine → ZnCO3


212. Answer (3) 219. Answer (1)
+7 CH2 – OH
2MnO4 – + 3H2O2 → n + n HOOC COOH
CH2 – OH
+4 Terephthalic acid
2MnO2 + 3O2 + 2H2O + 2OH

In basic medium MnO−4 is reduced to MnO2,


O
whereas in acidic medium it is reduced to Mn+2.
213. Answer (1) OCH2 – CH 2 – O – C C
n
CaCO3 + H2SO4 → CaSO4 + H2O + CO2 O
Dacron
214. Answer (3)
220. Answer (3)
Buna – N is a synthetic rubber which can be
Primary structure is unaffected by heating
stretched and retains its original status on
221. Answer (1)
releasing the force.
Mixture of chloroxylenol and terpineol is known
215. Answer (2)
as Dettol. It acts as Antiseptic.
DNA consists of -D-2-deoxyribose sugar
222. Answer (3)
whereas RNA consists of -D-ribose.
216. Answer (3) Photochemical smog contain:

Corporate Office : Aakash Tower, 8, Pusa Road, New Delhi-110005 [79]

For More Material Join: @JEEAdvanced_2024


For More Material Join: @JEEAdvanced_2024
Unique Practice Questions Chemistry

Ozone, nitric oxide, organic compounds, Nicotine — Pesticide


nitrogen dioxide, formaldehyde. Sulphate — Laxative effect
 SO2 is not the part of photochemical smog. Fluoride — Bending of bones
223. Answer (1) 230. Answer (2)
During the denaturation of proteins hydrogen Nylon 6, 6 → used in making bristles of brushes
bonds are disturbed. As a result, the secondary
Low density polythene → used in making Toys
and tertiary structures are destroyed but the
High density polythene → used in making
primary structures remain intact.
Buckets
224. Answer (2)
Drugs that bind to the receptor site and inhibit its Teflon → used in making non-stick utensils

natural function are called Antagonists. 231. Answer (2)

225. Answer (3) 6


CH2OH
5 1 O
Glycerol, on heating with KHSO4, undergoes O HOH2C
H H H
H
dehydration to give unsaturated aldehyde called 4 1 2 5
OH H O H HO
HO 3 2 3 4 CH2OH
acrolein. So, statement I is correct. 6
Glycosidic
H OH OH H
linkage
-D-Glucose -D-Fructose

Sucrose
Hence in sucrose glycosidic linkage between C1

Acrolein has piercing unpleasant smell. So, of -glucose and C2 of -D-fructose is found

statement-II is incorrect. Maltose  Glycosidic linkage between C1 and


226. Answer (3) C4
Micelle formation is an endothermic process with Lactose  Glycosidic linkage between C1 and
positive entropy change. C4
227. Answer (2)
Amylose  Glycosidic linkage between C1 and
Cast iron is made by melting pig iron with scrap C4
iron and coke using hot air blast. 232. Answer (2)
Hence Statement-I is incorrect Some drugs do not bind to the enzyme’s active
But Pig iron has relatively more carbon content site. These bind to a different site of enzyme
Hence statement-II is incorrect which is called allosteric site.
228. Answer (3) 233. Answer (1)
High purity (>99.95%) H2 is obtained by Urea is used as a stabilizer for the storage of
electrolysing warm aqueous Ba(OH)2 solution H2O2.
between nickel electrodes. 234. Answer (4)
229. Answer (3) Thin layer chromatography is a technique used
Sodium arsinite — Herbicide to isolate non-volatile mixtures.

Corporate Office : Aakash Tower, 8, Pusa Road, New Delhi-110005 [80]

For More Material Join: @JEEAdvanced_2024


For More Material Join: @JEEAdvanced_2024
Unique Practice Questions Chemistry

Hence, mixture of p-nitrophenol and Picric acid Penicillin – G is a narrow spectrum antibiotic.
is separated by TLC. Salvarsan is mainly active against spirochete, a
235. Answer (4) bacteria that causes syphilis
Terpineol is an antiseptic.
241. Answer (3)

More polar gases easily adsorbs on activated


charcoal.
The product do not contain any ion in their And more polar gases has more (higher) critical
constitution hence it is a non-ionic detergent. temperature as compared to non-polar or less
236. Answer (1) polar gases.
Finest gold sol is red in colour; as the size of  Gases with higher critical temperature
particles increases, it appears purple, then blue adsorbed more.
and finally golden. 242. Answer (1)
The colour of colloidal solution depends on the
Leaching and calcination are the processes
wavelength of light scattered by the dispersed
which are involved in the extraction of the
particles. The wavelength of light further
metals.
depends on size and nature of the particles.
Liquation, Electrolytic refining, Distillation are
Hence, Both A and R are true and R is the
used in the refining or purification of metal.
correct explanation of A
243. Answer (2)
237. Answer (1)
The correct match of pollutants and disease
Pollutant Source
because of the excess of these pollutants are:
Microorganisms → Domestic sewage
Sulphate ⟶ Laxative effect
Plant nutrients → Chemical fertilizers

Toxic heavy metals → Chemical factory Nitrate ⟶ Methemoglobinemia

Sediment → Strip mining Lead ⟶ Kidney damage


238. Answer (3)
Fluoride → Brown mottling of teeth
When a mixture of isoprene and sulphur is
244. Answer (2)
heated, isoprene gets polymerised to natural
rubber and then vulcanization of natural rubber
with sulphur takes place. Polymers Commercial
239. Answer (3) names
Animal starch is the other name of glycogen
A. Phenol- Novolac
because its structure is similar to amylopectin.
formaldehyde
240. Answer (01.00)
resin
Ofloxacin is the only broad spectrum antibiotic
given in the question

Corporate Office : Aakash Tower, 8, Pusa Road, New Delhi-110005 [81]

For More Material Join: @JEEAdvanced_2024


For More Material Join: @JEEAdvanced_2024
Unique Practice Questions Chemistry

B. Copolymer of Buna-S Natural rubber Isoprene

1,3-butadiene and 249. Answer (4)

styrene The correct match is

(A Chloroform & (III) Simple distillation


C. Polyester of glycol Glyptal
) Aniline
and phthalic acid
(B Benzoic acid & (IV) Crystallisation
D. Polyester of glycol Dacron
) Napthalen (Sublimation
and terephthalic
e is not used
acid
as both
 The Correct match is sublime

A – II; B – III, C – I ; D – IV heating)

245. Answer (3) (C Water & Aniline (I) Steam distillation

• A is morphine which is a narcotic analgesic. )

• B is chloroxylenol, an antiseptic. (D Napthalene & (II) Sublimation (only


) Sodium naphthalene
• C is Nardil, an antidepressant.
chloride has the
• D is saccharin, which is around 550 times
tendency for
sweeter than cane sugar.
sublimation)
246 Answer (3)
Parchment paper is a semi-permeable
250. Answer (04.00)
membrane which allows particles of true solution
to pass through as their size are too small. The principal ores of iron are :

Assertion is correct but reason is incorrect. Siderite, Haematite, Magnetite, Limonite.


247. Answer (1) 251. Answer (4)

Both Statement are correct. • Clean water would have BOD value of less
• Fuel obtained from plastic waste has high than 5 ppm whereas highly polluted water
octane rating. It contain no lead and is could have BOD value of 17 ppm or more.
known as “green fuel”. • Eutrophication results in decrease in the
• The non bio-degradable fly ash and slag amount of dissolved oxygen.
from steel industry can be used by cement 252. Answer (3)
industry.
248. Answer (1)
Polymer Monomer
Neoprene Chloroprene
Teflon CF2 = CF2
Acrilan (PAN) Acrylonitrile

Corporate Office : Aakash Tower, 8, Pusa Road, New Delhi-110005 [82]

For More Material Join: @JEEAdvanced_2024


For More Material Join: @JEEAdvanced_2024
Unique Practice Questions Chemistry

253. Answer (3) spread over a glass plate of suitable size and act

Drugs can inhibit the attachment of substrate on as an adsorbent.

active site of Enzyme in two ways. 258. Answer (1)


Albumin from the egg was poured into 100 mL
(1) Competitive, (2) Non-competitive
of 5% (w/v) NaCl solution in water. This would
Competitive inhibitors bind on the active site of result in the formation of lyophilic sol. Albumin
Enzymes. Non-Competitive inhibitors bind on molecules get dispersed in water the colloidal
allosteric site. particles of albumin are stabilised by hydrogen
254. Answer (2) bond with water molecules.
Magnesium alloys are used to make body of 259. Answer (1)
aircraft which is lightweight and resistant to The term “gangue” is used for earthy or
corrosion. Calcium is responsible for cell undesired materials in ore.
membrane integrity. 260. Answer (4)
255. Answer (2) Aldrin and Dieldrin are examples of pesticides
CH2 – C = CH – CH 2 whereas Sodium chlorate (NaClO3) and Sodium
(A) Elastomer
Cl arsinite (Na3ASO3) are examples of herbicides.
n
261. Answer (2)
H H O O
(B) Fibers Seldane → Antihistamine
N – (CH2)6 – N – C – (CH 2)4 – C
n
Amytal → Barbiturate (Hypnotic)
Cl
Thermoplastic polymer Aspartame → Artificial sweetener
(C)
CH2 – CH
n Prontosil → Antibiotics
OH OH 262. Answer (01)
CH2 CH2
(D) Thermosetting In a linear tetrapeptide, four amino acids are
polymer
n linked and three peptide bonds are present.
256. Answer (4) Hence, 4 – 3 = 1
Action of an enzyme can be altered by a number 263. Answer (4)
of factors like temperature, pH, presence of Rayon is not natural polymer. It is semi-
activators and coenzymes and presence of synthetic, rest all are natural polymers
inhibitors and poisons. 264. Answer (4)

Inhibitors or poisons interact with the active Amylose is a linear polymer formed by
functional groups on the enzyme surface and combination of -D glucose through 1, 4-
often reduce or completely destroy the catalytic glycosidic linkage.
activity of the enzymes. It is water soluble
257. Answer (1)
So, assertion is incorrect
Thin layer chromatography is an adsorption
265. Answer (3)
chromatography. A thin layer of silica gel is

Corporate Office : Aakash Tower, 8, Pusa Road, New Delhi-110005 [83]

For More Material Join: @JEEAdvanced_2024


For More Material Join: @JEEAdvanced_2024
Unique Practice Questions Chemistry

 O
HPh H + Ph
(Phenolphthalein) (Ionised form) C
(unionised form) (pink) O
(colourless) C

In basic medium, [H ] decreases & therefore

more of (Ph ) is produced OH


OH
(Phenolphthalein)

❑ ❑ ❑

Corporate Office : Aakash Tower, 8, Pusa Road, New Delhi-110005 [84]

For More Material Join: @JEEAdvanced_2024

You might also like